Vous êtes sur la page 1sur 16

UROLOGA

Preparacin Examen de Seleccin 05/06 1 Vuelta


Pregunta 1.- R: 3
De los microorganismos referidos, el nico que es grampositivo es
el Streptococcus faecalis, hoy en da denominado Enterococcus faecalis.
El E. faecalis es el causante del 20% de las sepsis de origen urinario.
Se relaciona principalmente con infecciones urinarias en sujetos con
anomalas de la va urinaria y tambin con la produccin de endocarditis en sujetos sometidos a exploraciones invasivas, sobre el tubo
digestivo o aparato genitourinario.
El tratamiento recomendado para las ITU ocasionadas por este
germen estara constituido por el empleo de la ampicilina, pero dado
el aumento de resistencias, se aconseja asociar un aminoglucsido
que potencie su efecto. En caso de resistencia a ampicilina, el antibitico de eleccin sera un glucopptido, como la vancomicina.
El resto de los microorganismos son bacilos gramnegativos, que son
agentes causantes de la mayora de las ITU. De ellos, sin duda el ms
frecuente es E. Coli, responsable del 80% de las ITU.

Comentarios TEST

Pregunta 2.- R: 3
E. coli es el agente responsable con mayor frecuencia de las prostatitis en sujetos tanto mayores como menores de 35 aos (respuesta 1
cierta).
La respuesta 2 tambin es correcta, ya que Staphylococcus saprophyticus ocasiona entre el 10-15% de las ITU en mujeres jvenes, lo que le
convierte en el segundo microorganismo causal de infecciones urinarias en mujeres jvenes sexualmente activas tras E. coli.
La respuesta 3 es falsa, puesto que el agente causal ms frecuente
de ITU en varones a cualquier edad es E. coli. Proteus mirabilis es poco
frecuente, y se le relaciona con ITU en pacientes con patologa
obstructiva o sometidos a manipulacin instrumental. Dada su capacidad para hidrolizar la urea (ureasa positivo), ocasiona litiasis de fosfato
amnico magnsico.
P. aeruginosa causa infecciones urinarias fundamentalmente en pacientes hospitalizados, siendo efectivamente el antibitico de eleccin la ceftazidima (otras alternativas: piperacilina-tazobactam, imipenem, aztreonam, cefepima o ciprofloxacino).
Por ltimo, la respuesta 5 tambin es correcta, en diabticos las
infecciones por hongos son ms frecuentes que en la poblacin general. El tratamiento de la candidiasis urinaria sera fluconazol 200 mg/
24h, durante 7-14 das.
Pregunta 3.- R: 2
Las infecciones urinarias son ms frecuentes en la mujer que en el
varn a todas las edades; ello es debido fundamentalmente a la anatoma de la uretra femenina, ms corta que la del varn y cercana al
introito vaginal y anal. Por ello, la respuesta 2 es la falsa.
La nica excepcin se da en el perodo neonatal, donde la ITU es
ms frecuente en el nio que en la nia; esto se explica por la mayor
incidencia de anomalas congnitas de la va urinaria en el recin
nacido varn.
En el varn anciano, la incidencia de ITU es mayor que en el varn
joven, y esto se debe a la hipertrofia benigna de prstata, que provoca
dificultad para vaciar la vejiga y, por lo tanto, residuo postmiccional
que facilita las infecciones urinarias. Por otro lado, la mayor necesidad de sondaje vesical en este grupo de edad tambin est relacionada con el aumento en la incidencia de ITU. Sin embargo, la ITU en la
vejez sigue siendo ms frecuente en la mujer que en el varn, aunque
la diferencia en la incidencia sea menor que en los individuos jvenes.
Por otra parte, las infecciones nosocomiales ms frecuentes son las
urinarias, causadas fundamentalmente por bacilos gramnegativos (E.
coli) y asociadas a sondaje vesical (respuestas 4 y 5).
Pregunta 4.- R: 2
Durante el primer trimestre del embarazo se debe realizar al menos
un urocultivo, y si este es positivo, se debe tratar a la mujer, constituyendo una de las indicaciones de tratamiento de bacteriuria
asintomtica. Se detecta infeccin urinaria entre el 2 y 8% de las
mujeres embarazadas, siendo E. coli el microorganismo ms frecuentemente relacionado (respuesta 2 falsa). El tratamiento de eleccin
est constituido por fosfomicina (bacteriuria asintomtica), penicilinas y cefalosporinas (infeccin tracto urinario superior). Adems, el
M exico A rgentina
C hile U ruguay

Seguimiento a distancia

20-30% de las embarazadas con bacteriuria sintomtica termina sufriendo pielonefritis. Esta predisposicin a las infecciones altas durante la gestacin obedece a la disminucin del tono ureteral, al menor
peristaltismo ureteral y a la insuficiencia temporal de las vlvulas
vesicoureterales. Las ITU durante el embarazo, en especial las de las
vas altas, elevan el nmero de lactantes de bajo peso al nacer, los
partos prematuros y la mortalidad neonatal.
Pregunta 5.- R: 4
Nos encontramos ante una mujer joven con la tpica clnica de
pielonefritis aguda. Ante la sospecha clnica debemos solicitar un
hemograma y bioqumica sangunea, sistemtico de orina con test de
embarazo y una prueba de imagen. El dato clave en este caso clnico
es que el test de embarazo es positivo, lo cual va a influir en nuestra
actitud diagnstico-teraputica. Por ello la respuesta 1 es incorrecta,
ya que no podemos realizar radiografa simple de abdomen a una
mujer embarazada. Asimismo tambin es falsa la respuesta 2, puesto
que si bien la ampicilina se puede emplear durante la gestacin, no
as la gentamicina, por sus efectos oto y nefrotxicos. Tampoco se
pueden administrar quinolonas (afectan al cartlago de crecimiento),
ni cotrimoxazol (produce bloqueo secuencial del cido flico) (respuestas 3 y 5). Por lo tanto, la respuesta 4 es la correcta. Durante al
menos las primeras 24 horas, la paciente debe estar en observacin
para evaluar la respuesta al tratamiento antibitico, que se suele administrar por va parenteral, siendo las cefalosporinas de primera eleccin en este caso.
Pregunta 6.- R: 2
La cistitis aguda representa la infeccin urinaria ms frecuente. La
clnica consiste en sndrome miccional de intensidad variable, junto
con molestias hipogstricas y ocasionalmente febrcula (respuesta 5 falsa). En cuanto se tiene la sospecha clnica de cistitis aguda se debe
iniciar el tratamiento antibitico de forma emprica, sin esperar al resultado del antibiograma (respuesta 1 incorrecta), por diversos motivos. En
primer lugar, aproximadamente el 80% de las ITU bajas estn causadas
por E. coli, con lo cual est justificado administrar antibiticos que cubran dicho microorganismo de manera emprica. Por otro lado, si retrasamos la instauracin del tratamiento antimicrobiano, la infeccin puede
ascender y producir una infeccin urinaria alta, de mayor gravedad y
con mayor repercusin clnica.
La duracin recomendada de la antibioterapia en la cistitis aguda
no complicada es de tres das. En caso de presentar algn factor de
complicacin se debe prolongar el tratamiento a una semana (respuesta 2 correcta). El hecho de que se presente en un varn convierte
la cistitis en complicada (respuesta 4 incorrecta). Adems se considera
cistitis recurrente la presencia de ms de dos infecciones cada seis
meses (respuesta 3 errnea), en cuyo caso estara justificado realizar
profilaxis antimicrobiana.

Pregunta 6. Diagnstico diferencial de las ITU.


ITU baja

ITU alta (PNA)

Sndrome miccional

Fiebre alta

No

Afectacin del estado general

No

Dolor lumbar

No

Leucocitosis

No

VSG

No

Pregunta 7.- R: 3
Nos preguntan por el tratamiento antibitico de una cistitis aguda
no complicada. Los microorganismos ms frecuentes son bacilos
gramnegativos, fundamentalmente E. coli, por lo que el antibitico
debe incluir dichas bacterias en su espectro de actividad. Amoxicilina-clavulnico (respuesta 1) cumple con dicho criterio, administrndose en dosis de 500/125 cada 8 horas, durante tres das. Las fluoro-

CTO Medicina C/ Nez de Balboa, 115 28006 MADRID (Espaa) Tfno.: (91) 782 43 32 / Fax: (91) 782 43 27
E-mail: secretaria@ctomedicina.com; iberocto@ctomedicina.com WEB: www.ctomedicina.com; www.iberocto.com

UR Pg. 1

UROLOGA

Seguimiento a distancia

Preparacin Examen de Seleccin 05/06 1 Vuelta

Pregunta 8.- R: 5
En este caso clnico nos hacen referencia a un cuadro denominado pielonefritis enfisematosa. Es tpico de diabticos, fundamentalmente mujeres, acompandose a menudo de obstruccin urinaria e
infeccin crnica. E. coli provoca la mayora de los casos, aunque a
veces se aslan otras enterobacterias. Se caracteriza por una evolucin
clnica de progresin rpida, con fiebre alta, leucocitosis, puopercusin renal positiva, palpndose a veces empastamiento en la zona
lumbar, piuria y glucosuria, junto con necrosis del parnquima renal
y acumulacin de gases de fermentacin en el rin y en los tejidos
perinfricos. Precisamente este hecho provoca que en las pruebas de
imagen (radiografa simple de abdomen, ecografa, TAC) se visualice
gas en el parnquima renal, constituyendo un dato que nos debe
hacer sospechar que nos encontramos ante esta entidad clnica. El
paciente puede presentar una evolucin fatal en pocas horas, por lo
que el tratamiento de eleccin es la nefrectoma de urgencia, junto
con la administracin de antibiticos de amplio espectro por va
sistmica.
Pregunta 9.- R: 4
En esta pregunta nos encontramos ante un paciente que presenta
una prostatitis aguda. Dicho cuadro es tpico de varones en edad
media de la vida. Cursa con malestar general, quebrantamiento, fiebre muy elevada, sndrome miccional (que suele ser muy intenso),
dolor perineal y perianal. A la exploracin fsica, en el tacto rectal la
prstata es dolorosa, est aumentada de tamao, congestiva y caliente. Debemos realizar hemograma, analtica y cultivo de orina, toma de
constantes vitales (este cuadro puede provocar sepsis urolgica), pero
nunca masaje prosttico para obtener lquido para anlisis, puesto
que podemos empeorar la bacteriemia del paciente (respuesta 4 incorrecta). Asimismo tambin est contraindicado en estas circunstancias el sondaje vesical, por lo que si el paciente presenta una retencin urinaria en el contexto de una prostatitis aguda, se debe colocar

una talla vesical, tambin denominada cistostoma suprapbica, para


evacuar la orina.
Los dos antibiticos que mejor penetran el tejido prosttico son las
fluoroquinolonas y el trimetoprim-sulfametoxazol, los cuales presentan, adems, buena actividad frente a E. coli. La duracin de la
antibioterapia debe prolongarse durante cuatro semanas, para evitar
el acantonamiento del microorganismo y que se produzcan prostatitis
crnicas bacterianas (ver tabla a pie de pgina).
Pregunta 10.- R: 5
En este caso clnico se nos describe la tpica presentacin de una
orquiepididimitis aguda. En individuos menores de 35 aos los
microorganismos ms frecuentemente implicados son de transmisin sexual (fundamentalmente Chlamydia trachomatis), mientras que
en mayores de 35 aos el principal agente causal es E. coli (respuesta
4 falsa). Cursa con fiebre, dolor de inicio insidioso, a la exploracin
el teste presenta signos inflamatorios, estando la polaridad conservada y alivindose el dolor al elevar el testculo, a diferencia de lo que
ocurre en la torsin testicular (respuesta 2 incorrecta).
La leucocitosis en estos pacientes suele ser muy elevada, en general
mayor de 20.000 leucocitos/mm3.
El tratamiento depende de la edad. Si sospechamos agentes de
transmisin sexual, el tratamiento de eleccin es doxiciclina 100 mg/
24h durante 10 das (cubre C. trachomatis), asocindose 250 mg de
ceftriaxona en dosis nica para cubrir el gonococo. En pacientes
mayores de 35 aos, el tratamiento debe ser una quinolona durante
2-3 semanas (respuesta 1 falsa), siendo conveniente aadir antiinflamatorios los primeros das del tratamiento (respuesta 5 correcta). Slo
en casos de orquiepididimitis de evolucin trpida, que no responden al tratamiento instaurado, en los cuales se produce abscesificacin del teste, estara justificada la exploracin quirrgica del testculo
(respuesta 3 incorrecta).
Pregunta 11.- R: 5
Los pacientes sondados presentan una mayor incidencia de infeccin urinaria (respuesta 5 falsa). Los factores asociados al mayor riesgo de
infeccin por sonda comprenden el sexo femenino, el sondaje prolongado, una enfermedad de base grave, la desconexin de la sonda del
tubo de drenaje u otros defectos de las sondas. Se produce infeccin
cuando las bacterias llegan a la vejiga por una de estas dos vas: por
migracin a travs de la columna de orina en la luz de la sonda (va
intraluminal) o por el ascenso desde la sonda a travs de la mucosa (va
periuretral). Las bacterias normalmente entran en la sonda a travs de la
unin entre la sonda y el tubo de drenaje. Por ello, a los pocos das de
colocar una sonda el individuo presenta bacteriuria (respuesta 1).
Desde una perspectiva clnica, la mayora de las infecciones asociadas a sondas apenas inducen sntomas, no se acompaan de fiebre
y a menudo remiten con su retirada, no siendo necesario el trata-

Pregunta 9. Diagnstico diferencial de procesos prostticos.


CLNICA

H
ITU

CULTIVO
ORINA

Prostatitis
aguda

E. coli

Cuadro sptico

P. crnica
bacteriana

E. coli

Irritativo con
reagudizaciones, sin fiebre ni
leucocitosis

+/+ en
reagudizaciones

>10 l/campo

P. crnica
abacteriana

Ureaplasma,
Mycoplasma

Cronicidad, empeoramiento
(no fiebre ni malestar
general, slo empeoran del
sndrome miccional)

> 10 l/campo

Crnica y oscilante

< 10 l/campo

Prostatodinia No infecciosa?

Pg. 2 UR

M exico A rgentina
C hile U ruguay

LQUIDO
PROSTTICO

CULTIVO
LQUIDO
PROSTTICO

ETIOLOGA

Nunca hacer masaje prosttico


ni sondaje.

TRATAMIENTO
Cotrimoxazol.
Fluorquinolonas 4
semanas.
Cotrimoxazol.
Fluorquinolonas
6-12 semanas.

Doxiciclina

Alfabloqueantes.
R elajantes
musculares.

CTO Medicina C/ Nez de Balboa, 115 28006 MADRID (Espaa) Tfno.: (91) 782 43 32 / Fax: (91) 782 43 27
E-mail: secretaria@ctomedicina.com; iberocto@ctomedicina.com WEB: www.ctomedicina.com; www.iberocto.com

Comentarios TEST

quinolonas, como el ciprofloxacino y el norfloxacino (respuestas 2 y


4) son de los antibiticos ms utilizados en las ITU, presentando buena actividad frente a E. coli, aunque su uso indiscriminado est provocando un aumento en el ndice de resistencias a dicho microorganismo. La nitrofurantona (respuesta 5) es un antibitico de bajo coste,
muy til en este tipo de infecciones, que se administra en dosis de 50100 mg/6 h durante tres das.
Finalmente, la ceftriaxona, una cefalosporina de tercera generacin, es un antimicrobiano con excelente actividad frente a bacilos
gramnegativos. Pero el hecho de que slo pueda administrarse por
va parenteral y su elevado coste hacen que sea, entre los propuestos, el antibitico menos indicado para el tratamiento de una cistitis
aguda no complicada. Su uso suele reservarse para ITU complicadas.

UROLOGA

Preparacin Examen de Seleccin 05/06 1 Vuelta


miento antibitico (respuesta 2). A veces se pueden evitar las infecciones urinarias de este tipo si se emplea un sistema colector cerrado y
estril, si se mantiene una asepsia total durante la colocacin y el
mantenimiento de la sonda, y si se toman medidas para reducir al
mnimo las infecciones cruzadas. La administracin de antibiticos
previo al cambio de sonda en individuos que precisan de sondaje
permanente evita bacteriemias relacionadas con la manipulacin. En
estos pacientes es preferible el empleo de sondas de silicona, que
duran dos meses (respuestas 3 y 4).

Comentarios TEST

Pregunta 12.- R: 4
El aparato genitourinario es el sitio ms frecuente de afectacin
extrapulmonar, de tal manera que el 5% de los pacientes con tuberculosis activa presentan afectacin del tracto genitourinario. El microorganismo que lo produce con mayor frecuencia es el Mycobacterium
tuberculosis (respuesta 5 falsa). Durante la fase de diseminacin
hematgena (primoinfeccin) se produce siembra de bacilos en ambos riones en el 90% de los pacientes. No obstante la enfermedad
clnica es unilateral, es decir, afecta a una sola unidad renal (respuestas 2 y 3 incorrectas).
El perodo de latencia entre la diseminacin hematgena y la afectacin clnica oscila entre 10 y 40 aos, afectando principalmente a
sujetos por debajo de 50 aos (respuesta 1 falsa).
El diagnstico definitivo se obtiene mediante el cultivo en medio
de Lowenstein, aunque deben obtenerse al menos tres muestras de
das diferentes, ya que el paso de bacilos a orina no es constante.
Finalmente, el tratamiento mdico es el mismo que el de la tuberculosis pulmonar, emplendose las mismas pautas de antibiticos. En
ocasiones es necesario el tratamiento quirrgico, sobre todo la
nefrectoma en casos de destruccin total del parnquima renal por
la tuberculosis (rin mastic).
Pregunta 13.- R: 4
En la tuberculosis genitourinaria el paso de orina infectada a travs
de la uretra prosttica puede llevar a la invasin de la prstata y a una
o ambas vesculas seminales. Sin embargo, a veces la lesin hematgena primaria en el tracto genitourinario se presenta en la prstata, y
puede ascender hacia la vejiga y despus descender al epiddimo
(respuesta 1 correcta). La afectacin clnica del rin se suele producir entre 10 y 40 aos despus de la primoinfeccin (respuesta 2
verdadera). Los pacientes suelen tener, por ello, una reaccin de
Mantoux positiva (respuesta 4 falsa). En el 70% de los pacientes los
sntomas son leves, siendo el ms frecuente la hematuria, dolor leve en
flanco o bien clico renal. No obstante, la afectacin vesical s produce una clnica llamativa, apareciendo un sndrome miccional intenso
rebelde al tratamiento, donde destaca una importante polaquiuria
(respuesta 3 correcta). En estos casos es caracterstico como hallazgo
de laboratorio la aparicin de una piuria cida con urocultivo estril.
En el varn es tpica la presencia de una orquiepididimitis crnica
que no responde al tratamiento antimicrobiano habitual. Adems, si
resultan afectados ambos conductos deferentes por el proceso tuberculoso, el individuo puede padecer esterilidad (respuesta 5 correcta).
Pregunta 14.- R: 5
La cistitis intersticial es una entidad que afecta en un 90% de los
casos a mujeres, la mayora de ellas en la edad media de la vida. Hoy
por hoy se desconoce su etiologa, aunque se cree que estn implicados factores neuroinmunoendocrinos.
Su diagnstico es de exclusin, existiendo una serie de criterios.
Clnicamente se caracteriza por un sndrome miccional crnico, donde es caracterstica la nicturia. El diagnstico diferencial se plantea con
cualquier patologa que cause un cuadro cisttico. En el caso de la
tuberculosis genitourinaria con afectacin vesical, tanto los medios de
tincin rpida (Ziehl) como el cultivo de Lowenstein nos ayudarn en
el diagnstico. En la cistitis aguda el crecimiento de un microorganismo
en el urocultivo y la respuesta al antibitico descartan una cistitis
intersticial, habida cuenta que adems este es un proceso crnico. En
una litiasis vesical las pruebas de imagen ya nos orientan a su diagnstico, siendo definitiva su visualizacin en la cistoscopia. Recordemos que
la presencia de lceras de Hunner y petequias submucosas son hallazgos cistoscpicos muy caractersticos de la cistitis intersticial. Un carciM exico A rgentina
C hile U ruguay

Seguimiento a distancia

noma in situ vesical suele cursar con citologas positivas, siendo la biopsia vesical el mtodo de diagnstico definitivo. Finalmente, una uretritis
aguda no causa un sndrome cisttico, sino que se presenta con flujo
uretral y disuria (respuesta 5).
Pregunta 15.- R: 3
La forma de presentacin clnica ms frecuente de una urolitiasis
es el llamado clico nefrtico o crisis renoureteral. Este puede ser
simple o complicado. Los factores que determinan que un clico
renal sea complicado son la presencia de obstruccin severa, fiebre,
dolor incontrolable a pesar del empleo de analgsicos en dosis mximas y rin nico.
En el caso clnico que nos ocupa, nuestro diagnstico de sospecha
debe ser ste, el de clico renal complicado, puesto que el paciente
presenta dolor lumbar y fiebre. Tras la anamnesis y exploracin fsica,
debemos realizar una radiografa simple de abdomen (respuesta 2),
que nos ayudar en el diagnstico (aunque la no visualizacin de
litiasis no descarta un clico nefrtico). Se deben tomar las constantes
vitales y obtener un hemograma y bioqumica sangunea, puesto que
el clico complicado puede derivar en una sepsis urolgica (respuestas 1 y 4). Asmismo debemos realizar una ecografa, para evaluar el
rin y el grado de uropata obstructiva que presenta (respuesta 5).
Por el contrario, no tiene sentido el sondaje vesical para descartar
uropata infravesical, ya que no hay datos que nos orienten a este
cuadro (imposibilidad para la miccin, masa suprapbica) y la probable uropata de este paciente sera supravesical por un clculo (respuesta 3 incorrecta).
Pregunta 16.- R: 5
La ecografa en este paciente detecta una uropata obstructiva causada por una litiasis radiotransparente. La orina retenida proximal a la
localizacin del clculo se infecta, y ello produce una serie de manifestaciones, la primera de las cuales suele ser la fiebre. De ah la importancia de la aparicin de fiebre en un clico nefrtico, que lo convierte, por definicin, en un clico complicado. Si el proceso evoluciona, el paciente puede sufrir una sepsis de origen urolgico, cuadro
muy grave. As ocurre en el caso que se nos presenta, donde el paciente sufre hipotensin, leucopenia (seal de que los mecanismos de
defensa del organismo frente a la infeccin estn comenzando a fallar) y anemia. En esta situacin es prioritario desobstruir el rin,
asegurando el drenaje de la orina. Existen fundamentalmente dos
mtodos para derivar la va urinaria: uno de forma retrgrada, el llamado catter doble J o pig-tail, y otro de manera antergrada, mediante la colocacin de una nefrostoma percutnea (respuesta 5 correcta). Por supuesto tambin se debe administrar tratamiento antibitico, pero, insistimos, lo prioritario es derivar el rin, ya que, en el
organismo, los antibiticos no son efectivos ante una cavidad sptica
no drenada.
Pregunta 17.- R: 3
Nos encontramos ante la tpica clnica de un clico nefrtico simple (no hay datos que sugieran un clico complicado). Se caracteriza
por un dolor muy intenso de inicio sbito, que aumenta y disminuye
de intensidad (tipo clico), irradindose desde la fosa lumbar hacia
los genitales externos, acompaado de sintomatologa vegetativa (nuseas, vmitos, sudoracin).
En la exploracin fsica la puopercusin renal suele ser positiva,
el abdomen puede ser doloroso, pero sin signos de peritonismo, y al
ser un dolor referido, los genitales externos no presentan alteraciones.
El diagnstico de este cuadro es fundamentalmente clnico. La radiografa simple de abdomen no es definitiva, puesto que la litiasis puede
ser muy pequea o radiotransparente (respuesta 2).
El tratamiento de este cuadro va encaminado a aliviar los sntomas
que presenta el paciente (respuesta 3 correcta). Para ello es preciso
disminuir la presin dentro de la va urinaria para calmar el dolor, lo
que suele conseguirse con la administracin de antiinflamatorios, generalmente en un primer momento por va parenteral (respuesta 4).
Tambin se pueden usar espasmolticos, que disminuyen la presin
intraureteral al relajar la pared del urter, y antiemticos si presenta
vmitos. Cuando el dolor es ms intenso no se debe forzar la ingesta
hdrica, ya que al aumentar la diuresis se incrementa la presin

CTO Medicina C/ Nez de Balboa, 115 28006 MADRID (Espaa) Tfno.: (91) 782 43 32 / Fax: (91) 782 43 27
E-mail: secretaria@ctomedicina.com; iberocto@ctomedicina.com WEB: www.ctomedicina.com; www.iberocto.com

UR Pg. 3

UROLOGA

Preparacin Examen de Seleccin 05/06 1 Vuelta

intraureteral (respuesta 1). Si el individuo comenzara con fiebre, lo


primero que deberamos sospechar es un clico complicado (respuesta 5 incorrecta).

Pregunta 17. Manejo del clico renal agudo.

Pregunta 18.- R: 4
El efecto teraputico del citrato potsico sobre la calculognesis
se debe al aumento de los citratos urinarios y a su accin alcalinizante,
interviniendo la combinacin de tres aspectos diferentes:
1) Capacidad para formar compuestos solubles con el in calcio en
la orina, con lo cual disminuye la concentracin del calcio libre
en orina.
2) Efecto inhibidor de la cristalizacin de oxalatos y fosfatos clcicos
(respuesta 1).
3) Eleva el pH urinario, con lo cual acta evitando la formacin de
clculos de cido rico o redisolviendo los ya formados (respuestas 2 y 3).
Existen una serie de factores que regulan la excrecin renal de
citrato, de los que el principal es el equilibrio cido-base. En situaciones de acidosis metablica, se facilita la entrada de citrato en la
clula tubular, incrementndose su proceso oxidativo en la mitocondria y disminuyendo el citrato urinario (respuesta 5). El diurtico
acetazolamida disminuye la excrecin renal de citrato, por lo que
est contraindicada su administracin conjunta (respuesta 4 incorrecta). Las dosis teraputicas recomendadas de citrato potsico varan entre 20 y 100 mEq/da.
Pregunta 19.- R: 2
La hiperoxaluria es una de las causas de litiasis de oxalato clcico
(ver tabla). Puede ser primaria o bien secundaria.
La hiperoxaluria primaria es muy poco frecuente, se debe a un
defecto enzimtico y se hereda de manera autosmica recesiva.
Son mucho ms frecuentes las hiperoxalurias secundarias, producidas por patologas que causan malabsorcin de cidos grasos.
Una de estas patologas la constituye la enfermedad de Crohn. En
esta enfermedad inflamatoria intestinal, la malabsorcin provoca

aumento de la grasa y bilis intraluminal. El calcio de la luz intestinal se


une con facilidad a la grasa, desembocando en un proceso de saponificacin. El calcio entrico, que en una situacin normal tendra
que unirse a oxalato, est disminuido. El oxalato no unido se absorbe con facilidad por un mecanismo de difusin. Este pequeo incremento en la absorcin de oxalato y la subsecuente excrecin urinaria aumenta de forma dramtica el producto de formacin de oxalato de calcio. El tratamiento en estos pacientes consiste en administrar
suplementos orales de calcio, que se une al oxalato intraluminal y
limita su absorcin.
La colestiramina, una resina de intercambio inico, tambin es
til, ya que restablece a nivel intestinal el equilibrio de absorcin de
cidos grasos y calcio.
Pregunta 20.- R: 3
En la mayor parte de las litiasis que precisan tratamiento, se
emplea la litotricia extracorprea por ondas de choque (LEOC)
como primera eleccin. Casi todos los clculos son susceptibles de
tratamiento mediante LEOC. La onda de choque produce fenmenos de compresin y descompresin sobre la litiasis, facilitando su
fragmentacin. Tres son las contraindicaciones absolutas al tratamiento con LEOC:
1) Embarazo, por los efectos secundarios que puede tener sobre el
feto.
2) Obstruccin distal a la localizacin de la litiasis, que impedira la
expulsin de los fragmentos litisicos.
3) Infeccin activa, en cuyo caso lo prioritario es derivar la va
urinaria.
La ciruga fue el tratamiento estndar hasta la aparicin de la
LEOC. Sin embargo, todava es necesario recurrir a tcnicas quirrgicas en ciertas ocasiones. Una de ellas la representan los clculos
coraliformes (respuesta 3 correcta). Otra indicacin sera en casos de
fracaso de LEOC, as como en pacientes con litiasis vesical. Las tcnicas quirrgicas pueden ser abiertas o bien endoscpicas, estas ltimas en auge hoy da. La eleccin de una u otra depende fundamentalmente de las caractersticas de la litiasis (forma, tamao, situacin)
y tambin de la experiencia de cada centro.
Pregunta 21.- R: 3
Las litiasis infectivas, de estruvita o fosfato amnico magnsico, tienen
una serie de caractersticas que las distinguen de otros tipos de litiasis.
Una de ellas es que son ms frecuentes en mujeres, y adems
tienen una mayor tendencia a formar clculos coraliformes o en asta
de venado (respuesta 3 correcta). Est causada por microorganismos
que hidrolizan la urea mediante la enzima denominada ureasa, aumentando el amonio urinario y produciendo un pH en la orina
alcalino. Dentro de las bacterias desdobladoras de la urea, la ms
importante es Proteus mirabilis, existiendo otras como Klebsiella, Pseudomonas, Serratia y Enterobacter.
Los cuerpos extraos (sondas vesicales, suturas, catteres ureterales) y la presencia de residuo postmiccional en la vejiga (por patologa prosttica, disfunciones neurolgicas vesicales) favorecen la formacin de este tipo de litiasis. El tratamiento antibitico se realizar
en funcin del urocultivo y antibiograma, logrando esterilizar la ori-

Pregunta 19. Causas de litiasis.


Hipe rcalciuria
Hiperuricosuria
idioptica
Etiologa

Hereditaria

Dieta

Diagnstico

Normocalcemia
e hipercalciuria

cido rico en
orina por
encima de los
valores
normales

Tratamie nto

Diurticos
tiacdicos

Alopurinol o
dieta

Pg. 4 UR

M exico A rgentina
C hile U ruguay

Hipe rparatiroidismo primario

Acidosis tubular
re nal distal

Tumoral

Hereditaria

Hipe roxaluria Hipe roxaluria


inte stinal
he re ditaria
Ciruga
intestinal

Acidosis
Hipercalcemia e
Ox alato en
hiperclormica con
hipofosfatemia con valores
niveles
pH urinario
invertidos en orina (aunque
superiores a lo
mnimo
la hipercalciuria es frecuente)
normal
>5,5
Ciruga (antes de que se
produzca la lesin renal)

R eposicin de
lcalis

Colestiramina,
carga oral de
calcio

Hipocitraturia

Litiasis clcica
idioptica

Hereditaria

Dieta,
hereditaria

Aumento de
ox alat o y
cidos
gluclico y
glicrico

Citrato en orina
<300 mg/dl

L q u i d o s ,
piridox ina

Suplementos de
lcalis

Tiacidas y
l q u i d o s

CTO Medicina C/ Nez de Balboa, 115 28006 MADRID (Espaa) Tfno.: (91) 782 43 32 / Fax: (91) 782 43 27
E-mail: secretaria@ctomedicina.com; iberocto@ctomedicina.com WEB: www.ctomedicina.com; www.iberocto.com

Comentarios TEST

Seguimiento a distancia

UROLOGA

Preparacin Examen de Seleccin 05/06 1 Vuelta


na slo durante su administracin.
El cido acetohidroxmico y cido propinico son inhibidores irreversibles de la ureasa que en ocasiones se emplean. Sin embargo, su uso
est limitado por los efectos secundarios que provocan, sobre todo a
nivel neurolgico (cefaleas, temblores) y vascular (trombosis venosas).

Comentarios TEST

Pregunta 22.- R: 4
Nos encontramos ante un caso de litiasis coraliforme completa en
una mujer de 65 aos, causndole clnica de pielonefritis aguda en
varias ocasiones, lo cual no es infrecuente en estas pacientes, dada la
naturaleza infectiva de dichas litiasis. Este clculo coraliforme ha causado una anulacin funcional de la unidad renal, observndose este
hecho en la urografa intravenosa realizada. Adems, en el renograma
isotpico con DMSA, la funcin relativa de dicho rin es de solamente el 15%.
Hay varios hechos que provocan que el tratamiento de eleccin
sea la nefrectoma (respuesta 4).
El que la litiasis sea coraliforme completa descarta la litotricia como
tratamiento (respuesta 1) y la ciruga percutnea (respuesta 5). Teniendo en cuenta adems que ese rin slo funciona a un 15%, los
intentos de preservar la unidad renal no tienen sentido, y ms en una
paciente de 65 aos. La antibioterapia profilctica prolongada no va
a evitar nuevas infecciones, favoreciendo adems la aparicin de resistencias (respuesta 3).
Si la paciente estuviera asintomtica se podra plantear la abstencin teraputica, pero ese rin est padeciendo pielonefritis de repeticin, lo que seguramente habr producido una pielonefritis
xantogranulomatosa (es un diagnstico anatomopatolgico). Por todo
ello, el tratamiento debe ser la nefrectoma.
Pregunta 23.- R: 2
La hematuria macro o microscpica es la forma de presentacin
clnica ms frecuente del cncer vesical (respuesta 1). Sin embargo, en
mujeres, la causa ms importante de hematuria la constituyen las
cistitis agudas (respuesta 2 incorrecta).
Como bien se expone en la respuesta 3, la causa ms frecuente de
hematuria en el varn mayor de 55 aos es la hiperplasia benigna de
prstata, pero antes debemos descartar que el paciente padezca un
tumor urotelial, mediante la anamnesis (sobre todo antecedente de
tabaquismo), pruebas de imagen (ecografa, urografa intravenosa) y
citologas en orina. Si tras la realizacin de estas pruebas tenemos el
diagnstico de sospecha de cncer vesical, deberemos someter al paciente a una exploracin bajo anestesia ms reseccin transuretral, la
cual tiene funciones diagnsticas (confirmacin anatomopatolgica) y
teraputicas (extirpacin endoscpica de las lesiones) (respuesta 4).
El tumor vesical ms frecuente es aquel que deriva del urotelio o
epitelio de clulas transicionales, el cual, adems de recubrir la vejiga,
tambin est presente en el tracto urinario superior (pelvis, clices y
urter). De ah que el tumor urotelial pueda asentar tanto en vejiga
como en el sistema urinario superior (respuesta 5).
Pregunta 24.- R: 3
Aunque la causa ms frecuente de hematuria vesical no es el cncer
de vejiga, este s es una de las causas ms graves. Esta hematuria est
presente en el 75-80% de los pacientes que tienen un tumor vesical,
puede ser macro o microscpica, intermitente ms que constante y en
un pequeo porcentaje se acompaa de sntomas de irritacin vesical.
Entre las pruebas de laboratorio destaca la analtica de orina (respuesta
4), la cual confirmar la existencia de hematuria (a veces lo que el
paciente refiere como orina con sangre en realidad es orina colrica
o concentrada). Tambin podemos hallar piuria ms bacteriuria, lo
que nos debe orientar a una causa infecciosa de la hematuria. La citologa
en orina tiene una elevada rentabilidad (respuesta 2), ya que las clulas
exfoliadas tanto del urotelio normal como del neoplsico pueden identificarse con facilidad en la orina. El examen de las muestras citolgicas
permite detectar el tumor, ya sea en el momento de la presentacin
inicial o durante el seguimiento. Asimismo, en el proceso de evaluacin
inicial de una hematuria debemos realizar una prueba de imagen,
generalmente una urografa intravenosa o bien una ecografa (respuestas 1 y 5). La cistoscopia no se incluye dentro de las pruebas de primera
lnea para el diagnstico de hematuria (respuesta 3 incorrecta), ya que
M exico A rgentina
C hile U ruguay

Seguimiento a distancia

es una exploracin agresiva, con co-morbilidad asociada, reservndose por lo tanto para casos en los cuales persistan dudas de la existencia
de tumor urotelial tras la realizacin de las pruebas de primera eleccin
ya comentadas.
Pregunta 25.- R: 2
El carcinoma epidermoide o escamoso constituye entre el 5 y el
10% del total de los tumores vesicales. Adems de los antecedentes
de infeccin crnica, clculos vesicales o portador de sonda permanente (respuesta 1), un factor de riesgo clsico es la infeccin por
Schistosoma haematobium, un parsito frecuente en zonas del norte
de frica y Medio Oriente, fundamentalmente en Egipto (donde el
carcinoma escamoso representa el 60% de los cnceres vesicales). El
adenocarcinoma vesical se produce en el 2% de los casos de neoplasia vesical, y est relacionado con la cistitis glandular y la extrofia
vesical. Tanto el adenocarcinoma como el carcinoma epidermoide
son tumores que suelen ser slidos e invasivos al diagnstico, y por lo
tanto de peor pronstico que el carcinoma urotelial (respuesta 2
incorrecta).
El carcinoma in situ es una lesin intraepitelial, muy anaplsica y
desdiferenciada, con clulas grandes de nuclolo prominente, estando la polaridad celular del urotelio alterada. Una de sus caractersticas es el ser tremendamente agresivo, progresando en muchos casos a
enfermedad invasiva (respuesta 3).
En la cistectoma radical que se realiza como tratamiento del cncer vesical infiltrante existen diversas tcnicas de derivacin urinaria.
Una de las ms utilizadas emplea un segmento de leon, al que se
anastomosan por un extremo los urteres y el extremo distal se aboca
a piel, formando un estoma urinario (se denomina derivacin tipo
Bricker). Este segmento intestinal absorbe los hidrogeniones de la orina, pudiendo producir acidosis metablica (respuesta 4).
En los tumores vesicales es imprescindible la realizacin de una
reseccin transuretral (RTU), que nos confirme el tipo de cncer vesical,
el grado de diferenciacin celular y el nivel de infiltracin del tumor
en las capas de la vejiga, ya que todo ello va a determinar el pronstico y nuestra actitud teraputica (respuesta 5).

Pregunta 25. Causas de tumores vesicales.


- Am inas arom ticas (2-naftilam ina):
industria textil, industria del
caucho, colorantes.
- Fenacetinas crnicas.
- Sacarina, ciclam ato.
- Ciclofosfam ida (acrolena).
- TABACO, ortofenoles, triptfano...

Transicionales (90%).
Mejor pronstico.

- Schistosom a haem atobium .


- Litiasis, infecciones, catteres.

Escamosos (8%).

- Cistitis glandular.
- Extrofia vesical.

Adenocarcinomas
(2%).

Pregunta 26.- R: 3
La urografa intravenosa (UIV) permanece como la prueba de imagen ms comn para evaluar la hematuria. Los tumores vesicales pueden reconocerse como pedunculados, defectos radiolcidos de llenado que se proyectan hacia la luz, los tumores infiltrantes pueden
fijar o aplanar la pared vesical. En la ecografa los tumores aparecen
como focos ecognicos que se proyectan hacia la luz. La invasin de
la pared vesical se reconoce cuando la pared vesical normal, que es
intensamente ecognica, se interrumpe por tejido tumoral menos ecognico. A la hora de evaluar los tumores vesicales, la ecografa posee
mayor sensibilidad que la UIV, pero sta evala mejor el tracto urinario superior (respuestas 1 y 2).
La citologa urinaria tiene una elevada rentabilidad en el diagnstico y seguimiento de los tumores uroteliales. Esta rentabilidad es ma-

CTO Medicina C/ Nez de Balboa, 115 28006 MADRID (Espaa) Tfno.: (91) 782 43 32 / Fax: (91) 782 43 27
E-mail: secretaria@ctomedicina.com; iberocto@ctomedicina.com WEB: www.ctomedicina.com; www.iberocto.com

UR Pg. 5

yor cuanto ms desdiferenciada y agresiva es la neoplasia. El tumor


vesical infiltrante es la forma ms agresiva de la enfermedad y la que
presenta peor pronstico, siendo las citologas en orina positivas en
un 90% de los casos (respuesta 3 incorrecta). El carcinoma in situ es
una lesin muy anaplsica, donde las citologas son positivas en el 8590% de los pacientes (respuesta 4).
El CT abdominoplvico se utiliza para caracterizar la extensin de
la invasin de la pared vesical, y sobre todo para detectar ndulos
linfticos plvicos en los casos de cncer vesical infiltrante. La presencia de adenopatas metastsicas en el TAC contraindica la cistectoma
radical con intencin curativa (respuesta 5).
Pregunta 27.- R: 5
El carcinoma urotelial o de clulas transicionales representa el 90%
de las neoplasias malignas vesicales. El carcinoma epidermoide o escamoso constituye el 8-10% y el adenocarcinoma menos del 2%
(respuesta 1).
El principal factor de riesgo para desarrollar un tumor urotelial del
tracto urinario superior es el tabaco. Asimismo, el abuso crnico de
analgsicos (sobre todo del tipo de fenacetinas) tambin se ha relacionado con este tipo de neoplasias (respuesta 2).
La clasificacin anatomopatolgica ms utilizada para estadiar el
cncer vesical es el TNM de 1997. En ella Ta (no infiltra la lmina
propia), T1 (invade la lmina propia) y Tis (carcinoma in situ, que
recordemos que es una lesin intraepitelial, respuesta 5 incorrecta),
suponen estadios superficiales de la enfermedad. En cuanto el tumor
invade la capa muscular de la vejiga (T2), se considera que el carcinoma es infiltrante (respuesta 4). T3 supone la afectacin de la grasa
perivesical y T4 la invasin de rganos vecinos (prstata, recto, hueso
ilaco). Estos tumores infiltrantes se comportan de manera agresiva,
produciendo metstasis en un 50% de los pacientes a los 2 aos del
diagnstico, siendo el pronstico infausto (respuesta 3).
Pregunta 28.- R: 3
El carcinoma in situ, al ser una lesin anaplsica y muy
desdiferenciada, constituye una variedad agresiva de cncer urotelial. Tiene una historia natural variable, pero muchos casos progresan a enfermedad invasiva. Por lo tanto su agresividad radica en la
elevada tendencia a la recidiva, y adems, cuando recidiva, lo hace
hacia formas metastatizantes (respuesta 1). El carcinoma in situ puede presentarse tanto cerca como lejos de una lesin exoftica o,
pocas veces, ocurrir en forma de lesiones focales o difusas en un
paciente sin tumores macroscpicos. En este ltimo caso, la clnica
que produce es la de un sndrome cisttico, generalmente sin hematuria (respuesta 4).
Los tumores superficiales (Ta, T1) suelen tener buen pronstico.
Sin embargo, si slo los tratamos con una reseccin transuretral, lo
ms probable es que recidiven, aunque la mayora de ellos no progresarn ni en grado ni en estadio (respuesta 2). La probabilidad de
recurrencia est marcada por una serie de factores, entre ellos la recidiva en el primer ao de la RTU (respuesta 5). Para evitar dichas recidivas
se aade quimioterapia intravesical, existiendo muchos tipos. De ellas,
la ms efectiva es la BCG (bacilo de Calmette-Guerin), que es el tratamiento de eleccin del carcinoma in situ, pero no de los tumores
infiltrantes (T2), donde el tratamiento es la cistectoma radical (respuesta 3 incorrecta).
Pregunta 29.- R: 1
Nos encontramos ante un caso clnico de un varn de 70 aos con
hematuria. Aunque no sea la causa ms frecuente, debemos descartar
que se trate de un tumor urotelial. El antecedente de tabaquismo es
muy importante, ya que el 60% de los individuos con un cncer
urotelial son fumadores. Adems, se cree que la exposicin ocupacional a ciertos productos qumicos tambin est relacionada (ha
trabajado en una industria textil).
En la ecografa realizada se evidencia una masa vesical de implantacin estrecha y aspecto papilar, por lo que probablemente corresponda a un tumor superficial. Por ello, habr que someter al paciente
a una reseccin transuretral. Antes, sin embargo, se debe evaluar el
tracto urinario superior mediante una urografa intravenosa, por si
tuviera un carcinoma urotelial de la va urinaria superior sincrnico
Pg. 6 UR

M exico A rgentina
C hile U ruguay

UROLOGA

Preparacin Examen de Seleccin 05/06 1 Vuelta

(respuesta 1 incorrecta).
La RTU es el nico medio para confirmar el tipo de tumor vesical y
el grado de infiltracin (respuesta 4). Si se confirman los hallazgos
ecogrficos, la RTU conseguir la extirpacin completa del tumor
superficial, siendo el pronstico de este tipo de neoplasias bueno
(respuestas 2 y 3). Las citologas urinarias en tumores superficiales de
bajo grado tienen menor rentabilidad diagnstica. sta aumenta cuanto
ms desdiferenciado y agresivo es el carcinoma (respuesta 5).
Pregunta 30.- R: 4
Tras la realizacin de la reseccin transuretral, la anatoma patolgica revela que se trata de un tumor superficial que invade la lmina
propia (T1) y de grado dos (G2).
G se refiere al grado de diferenciacin citolgica, de tal manera
que tumores grado 1 (G1) estn bien diferenciados, grado 2 moderadamente diferenciados y grado 3 (G3) pobremente diferenciados. La
frecuencia de invasin tumoral, recurrencia y evolucin se correlaciona en gran medida con el grado del tumor: se observa recidiva en
el 10-20% de tumores grado 1, 19-37% de grado 2 y 33- 67% de
grado 3 (respuesta 3).
Otros factores que tambin determinan la posibilidad de recidiva
son: presencia de mltiples implantes (respuesta 1), tamao tumoral,
recurrencia en los primeros doce meses tras la RTU.
El grado de infiltracin es otro factor importante: cuanto ms invasiva sea la neoplasia mayor probabilidad de recidiva (respuesta 2). El
tabaco es el factor de riesgo ms importante en la aparicin de un
carcinoma urotelial y si, adems, el paciente contina con el hbito
tabquico, la probabilidad de recidiva y progresin tumoral aumenta
(respuesta 5). Sin embargo, la localizacin de los implantes tumorales
en la vejiga no es un factor determinante de la posibilidad de
recurrencia (respuesta 4 incorrecta).
Pregunta 31.- R: 3
Tras la realizacin de la RTU, en el carcinoma urotelial se debe
realizar seguimiento peridico mediante cistoscopia ms citologas en
orina. En el caso que nos ocupa, el tumor vesical ha recidivado a los seis
meses, lo cual se confirma sometiendo al paciente a una nueva exploracin bajo anestesia ms reseccin transuretral (EBA + RTU). El resultado anatomopatolgico es de carcinoma urotelial moderadamente
diferenciado que infiltra la lmina propia (T1G2), asociado a carcinoma in situ (Tis) en biopsias aleatorias de la vejiga, tomadas por la visualizacin de lesiones eritematosas dispersas por la mucosa vesical, que nos
hacen sospechar la presencia de carcinoma in situ. La aparicin de
carcinoma in situ va a determinar la evolucin y el pronstico de este
paciente, y por lo tanto nuestra actitud teraputica.
El tratamiento de eleccin del carcinoma in situ es la quimioterapia intravesical con BCG (respuesta 3 correcta). Se suele administrar
una dosis semanal durante seis semanas. Se desconoce el mecanismo
exacto por el cual la BCG ejerce su efecto antitumoral, pero parece
estar mediado por el sistema inmunitario. Suele observarse ulceracin
de la mucosa y formacin de granulomas despus de la instilacin
intravesical. Los efectos colaterales tras la administracin de BCG son
relativamente comunes, aunque son raras las complicaciones graves.
La mayora de los pacientes experimentan algn grado de polaquiuria
y tenesmo vesical. Tambin es relativamente frecuente un cuadro pseudogripal transitorio. Efectos ms graves incluyen sepsis, neumonitis e
incluso fallecimiento.
Pregunta 32.- R: 1
Ante la evidencia de tumor vesical, debemos siempre someter al
paciente a una reseccin transuretral, para extirpar el tumor y enviarlo al anatomopatlogo, el cual nos confirmar el tipo de tumor vesical
y el grado de infiltracin (respuesta 1 correcta).
En la mayora de los casos concuerdan los hallazgos cistoscpicos
con los anatomopatolgicos. Es decir, en el caso que nos ocupa,
probablemente el patlogo nos confirmar que se trata de un tumor
superficial que no invade la capa muscular de la vejiga.
Una vez tengamos la evidencia anatomopatolgica, decidiremos
nuestra actitud. En el caso de un tumor superficial, aadiremos quimioterapia intravesical (mitomicina c, tiotepa, adriamicina) para disminuir el ndice de recurrencias. Deberemos realizar seguimiento

CTO Medicina C/ Nez de Balboa, 115 28006 MADRID (Espaa) Tfno.: (91) 782 43 32 / Fax: (91) 782 43 27
E-mail: secretaria@ctomedicina.com; iberocto@ctomedicina.com WEB: www.ctomedicina.com; www.iberocto.com

Comentarios TEST

Seguimiento a distancia

UROLOGA

Preparacin Examen de Seleccin 05/06 1 Vuelta


peridico del paciente con citologas y cistoscopia.
En el caso de que no se pueda realizar cistoscopia (edad avanzada,
tratamiento anticoagulante, mala tolerancia) el seguimiento se lleva a
cabo con citologas en orina ms ecografa. En todos los casos se debe
volver a evaluar el tracto urinario superior al cabo de los dos aos,
mediante la realizacin de una urografa intravenosa.

Comentarios TEST

Pregunta 32. Manejo de las neoplasias vesicales.

Pregunta 33.- R: 2
La BCG intravesical consigue un 60-80% de respuestas completas, lo
que la convierte en la teraputica conservadora ms efectiva para el
carcinoma in situ. Estudios aleatorizados han demostrado que la inmunoterapia con BCG reduce el riesgo de progresin de la enfermedad y
la necesidad de cistectoma, y prolonga la supervivencia en pacientes
afectos de carcinoma in situ. Diversos estudios realizados demuestran
que a corto plazo no se observa recurrencia o progresin a tumor
infiltrante, pero la mayora de los trabajos con mayor seguimiento demuestran que la tasa de respuesta disminuye con el tiempo, con tasas
de progresin del 15 al 37% tras 2-5 aos de seguimiento, de los cuales
la mayora morirn por tumor transicional.
El carcinoma in situ que reaparece como fracaso tras un tratamiento con BCG tiene un pronstico ms desfavorable, como es el
caso que nos ocupa. El riesgo de progresin a 5 aos de aquellos
pacientes con una respuesta incompleta es del 95%, mientras que
slo es del 19% en aquellos con una respuesta inicial completa. Por lo
tanto, la indicacin precoz de cistectoma radical ante la constatacin
de persistencia de carcinoma in situ tras tratamiento con BCG es obligatoria (respuesta 2 correcta).
Pregunta 34.- R: 4
En este caso clnico se nos presenta un varn joven con hematuria
macroscpica, con un dato clave, que procede de Egipto. En esta
zona del norte de frica la parasitacin por Schistosoma haematobium
es muy prevalente, asocindose a la aparicin de carcinoma epidermoide de vejiga, representando este tipo de tumor el 60% de las
neoplasias vesicales.
En este paciente, el tumor produce un importante defecto de replecin vesical en la urografa intravenosa, lo cual orienta hacia la agresividad de este tipo de carcinoma vesical. Ello se confirma en la anatoma
patolgica tras la reseccin transuretral, que da como resultado un
carcinoma epidermoide que infiltra la capa muscular de la vejiga. Dada
la agresividad de este tumor, el tratamiento de eleccin es la
cistoprostatectoma radical en el varn (respuesta 4 correcta) y en la
mujer la cistectoma con histerectoma ms doble anexectoma. En el
mismo acto quirrgico se lleva a cabo la derivacin de los urteres,
existiendo diversas tcnicas. En muchos casos, sin embargo, la enfermedad acaba metastatizando a pesar de la cistectoma.
M exico A rgentina
C hile U ruguay

Seguimiento a distancia

Pregunta 35.- R: 2
En este varn de 65 aos y fumador, que presenta hematuria y
sndrome miccional, debemos descartar un cncer vesical, aunque,
insistimos, no sea sta la causa ms frecuente.
Dentro de las pruebas realizadas, las citologas urinarias son positivas para carcinoma urotelial, lo que ya nos debe hacer sospechar la
existencia de un tumor urotelial.
Necesitamos una prueba de imagen que nos oriente en el diagnstico, y la urografa intravenosa nos va a evaluar tanto el tracto urinario
superior como la vejiga. Si sta es normal, debemos seguir buscando
el tumor vesical, dada la elevada especificidad de las citologas positivas; para ello realizaremos una prueba ms agresiva, que es una
cistoscopia. Si en ella no encontramos ninguna alteracin en la vejiga,
debemos llevar a cabo una exploracin bajo anestesia con toma de
biopsias aleatorias, intentando descartar una neoplasia urotelial mediante el estudio anatomopatolgico (respuesta 2 correcta). Aqu agotaramos las pruebas diagnsticas. Si las biopsias son positivas, dependiendo del tipo de tumor vesical y del grado de infiltracin, iniciaremos el tratamiento pertinente. Si son negativas, realizaremos seguimiento peridico del paciente.
Pregunta 36. - R: 3
Nos encontramos ante un caso de un varn de 65 aos, fumador y
que presenta hematuria macroscpica. Se realiza una urografa intravenosa donde se aprecia una uropata obstructiva del sistema excretor
izquierdo (capta contraste, pero no lo elimina), y una vejiga con defectos de replecin en cara lateral derecha, sugestivos de neoplasia vesical.
Adems, las citologas en orina son positivas para carcinoma urotelial.
Por ello, debemos estudiar tanto el sistema excretor izquierdo como la
vejiga. Mediante una pielografa retrgrada conseguimos este objetivo
(respuesta 3 correcta), ya que para realizar esta prueba de imagen,
previamente hay que colocar un catter ureteral mediante una
cistoscopia. Por lo tanto, realizamos una cistoscopia que nos permitir
visualizar bien la vejiga y confirmar si esos defectos de replecin se
deben a tumoraciones vesicales, y tras localizar el meato ureteral izquierdo, se procede a la cateterizacin del mismo. As, podremos introducir contraste por dicho catter y evaluar mejor el tracto urinario
superior izquierdo, donde existe una uropata obstructiva. Adems, se
pueden obtener citologas selectivas de dicho sistema excretor, por si
existiera una neoplasia urotelial de la va urinaria alta.
Pregunta 37.- R: 4
Se realiza en el mismo paciente la pielografa retrgrada, demostrndose la existencia de un defecto de replecin en la pelvis
renal izquierda, lo cual puede estar causado por una neoplasia
urotelial. Esta sospecha viene apoyada por el hecho de que las
citologas tomadas de forma selectiva de la orina del sistema excretor izquierdo son positivas para carcinoma urotelial. Por todo ello,
existen suficientes datos para afirmar que el sujeto padece un tumor urotelial sincrnico, localizado en la vejiga (no olvidar que
existan defectos de replecin en la cara lateral derecha vesical) y
en la pelvis renal izquierda. El tratamiento de los tumores uroteliales
del tracto urinario superior es la nefroureterectoma, incluyendo
el urter terminal que desemboca en la vejiga a nivel del meato
(rodete o pastilla vesical). Con ello extirpamos todo el sistema excretor. Adems, en el mismo acto quirrgico, se debe llevar a cabo
una reseccin transuretral para extirpar las neoformaciones vesicales
(respuesta 4 correcta).
Pregunta 38.- R: 1
La glndula prosttica se diferencia desde el punto de vista ecogrfico y anatomopatolgico en cuatro partes (Mc Neal):
Estroma fibromuscular anterior, no glandular.
Zona central.
Zona perifrica.
Zona transicional.
El 70-80% de los cnceres de prstata (CP) asientan sobre la zona
perifrica y solamente un 10-20% de los CP se originan en la zona de

CTO Medicina C/ Nez de Balboa, 115 28006 MADRID (Espaa) Tfno.: (91) 782 43 32 / Fax: (91) 782 43 27
E-mail: secretaria@ctomedicina.com; iberocto@ctomedicina.com WEB: www.ctomedicina.com; www.iberocto.com

UR Pg. 7

transicin.
En la ciruga de la HPB (Adenomectoma y Reseccin Transuretral
Prosttica) la parte prosttica a extirpar es la zona de transicin, lugar
de asiento tpico de la HPB. Es por ello que al quedar la zona perifrica no estamos impidiendo el desarrollo posterior de un CP (respuesta 1 falsa).
La glndula prosttica presenta un crecimiento lento desde el
nacimiento hasta la pubertad, a partir de la cual sufre un crecimiento rpido, alcanzando un peso normal de aproximadamente
25 g. Este peso se mantiene estable hasta los 35 aos, cuando
comienza a apreciarse la aparicin de datos histolgicos de HPB
segn autopsias. No es hasta aproximadamente los 45 aos cuando comienzan los primeros datos clnicos de HPB (respuestas 2 y 3
ciertas).
La HPB es una patologa inherentemente asociada a la edad y a la
presencia de testculos funcionantes (no se constata la presencia de
HPB en sujetos castrados antes de la pubertad). La HPB se presenta
en mayor o menor medida en todo varn por encima de los 70
aos. El aumento de la esperanza de vida lleva aparejado un incremento en la prevalencia de esta enfermedad. Se estima que el 10%
de la poblacin masculina deber ser sometido a ciruga como tratamiento de la HPB, y que este porcentaje se ir incrementando con
el persistente incremento del envejecimiento poblacional (respuesta
4 cierta).
Se considera que tras la realizacin de ciruga de la HPB existe un
10% de piezas con hallazgos incidentales de CP, que se correlacionaran con estadios T1a y T1b. Es decir, son tumores originados
en la zona de transicin que pasan inadvertidos a la realizacin de
un tacto rectal. Recordemos que con el tacto rectal slo se palpa la
zona perifrica. El estadio T1a supone una clara indicacin de observacin y seguimiento, mientras que en el estadio T1b, esta actitud, aunque vlida, depender de otros factores: la esperanza de
vida del sujeto, el volumen tumoral, el Gleason del tumor, etc...
(respuesta 5 verdadera).
Pregunta 39.- R: 1
El concepto de HPB engloba, por un lado, el crecimiento de la
zona transicional, y por otro la sintomatologa derivada del obstculo mecnico que este ocasiona al flujo miccional. No obstante,
esta resistencia al flujo urinario no se encuentra en relacin directa
con el tamao prosttico. De hecho, se puede constatar la existencia de prstatas grandes con escasa repercusin obstructiva y prstatas pequeas de crecimiento intrauretral que provocan
precozmente obstruccin y la sintomatologa derivada de sta (respuesta 1 falsa).
En la uretra proximal existe fibra muscular lisa que responde a
estmulos simpticos (receptores alfa-1) produciendo un aumento o
disminucin del calibre urinario. Esto es considerado un factor dinmico, susceptible de modificacin farmacolgica. La administracin
de alfabloqueantes produce una relajacin de la fibra muscular lisa
de la uretra prosttica, aumentando el tracto de salida urinario, por lo
que son empleados en el tratamiento mdico de la HPB (respuesta 2
verdadera).
En la fisiopatologa de la HPB se distinguen tres fases:
Fase de compensacin: La obstruccin al flujo de salida de la
orina se supera gracias a una hipertrofia del msculo detrusor que
consigue vencer las altas presiones de vaciado (respuesta 3 verdadera). En esta fase existe poca clnica o es inexistente. Histolgicamente comienzan a aparecer celdas y trabeculaciones en el interior de la vejiga y posibilidad de herniaciones de la mucosa vesical
a travs de las fibras musculares (divertculos).
Fase clnica: La elongacin de las fibras musculares necesaria para
vencer la obstruccin que representa la uretra prosttica las lleva a
perder capacidad contrctil de manera progresiva. Esto justifica la
aparicin de sintomatologa tal como: retraso en el inicio de la miccin, prdida de la fuerza y de calibre miccional, chorro entrecortado y existencia de cierto grado de residuo postmiccional. De otro
lado, la hipertrofia del msculo detrusor condiciona una falta de
adaptacin a los cambios volumtricos vesicales, justificando as la
clnica de inestabilidad vesical que presentan estos pacientes,
traducindose en la aparicin de polaquiuria y nicturia (sntomas
Pg. 8 UR

M exico A rgentina
C hile U ruguay

UROLOGA

Preparacin Examen de Seleccin 05/06 1 Vuelta


irritativos (respuesta 4 verdadera).

Pregunta 39. Fisiopatologa de la HPB.


FASE DE
COMPENSACIN
- Disuria inicial.
- Alargamiento del
tiempo de miccin.
- Disminucin de la
fuerza del chorro.

FASE CLNICA
- Polaquiuria.
- Nicturia.
- Goteo terminal.
- Molestias en
hipogastrio.
- Imperiosidad.
- Hematuria.
- R iesgo de infeccin urinaria.

FASE DE
DESCOMPENSACIN
- Miccin por rebosamiento (incontinencia
paradjica).
- R etencin aguda de
orina.
- Hematuria.

Fase de descompensacin: Se produce una claudicacin del msculo detrusor que se traduce clnicamente en la aparicin de:
1. Retencin aguda de orina.
2. Miccin por rebosamiento: la vejiga se convierte en un mero
almacn no distensible. Cuando la presin intravesical supera
el punto de fuga, la orina se escapa.
3. Ureterohidronefrosis bilateral e insuficiencia renal aguda de causa obstructiva. Todo ello secundario a la prdida del mecanismo antirreflujo vesico-ureteral.
La evaluacin bsica de un paciente con HPB comprende la realizacin de:
Anamnesis e Historia clnica.
Exploracin fsica (tacto rectal).
Analtica de sangre (creatinina).
Pruebas complementarias:
1. Ecografa reno-vesico-prosttica (litiasis, tumor vesical,
ureterohidronefrosis, residuo postmiccional...).
2. Flujometra.
3. Ecografa transrectal prosttica (slo en el caso de que se prevea
ciruga).
La realizacin de la UIV se encuentra hoy en da cuestionada para
la evaluacin de un paciente prosttico, pues no aporta mayor rentabilidad diagnstica que la ecografa, salvo en aquellos casos que se
sospeche tumor del tracto urinario superior, donde estara indicada
su peticin para descartar esta posibilidad (prostticos con hematuria
macroscpica -respuesta 5 verdadera-).
La peticin de PSA estara indicada para descartar cncer prosttico, pero no para diagnstico de HPB.
Pregunta 40.- R: 2
En los acinos prostticos obstruidos, las clulas epiteliales sufren una
atrofia y degeneracin. Estos conglomerados de clulas degeneradas se
conocen con el nombre de cuerpos amilceos y seran el origen de los
clculos prostticos, ya que estn formados casi a partes iguales por
fosfato clcico y componentes orgnicos. Generalmente se localizan
en la zona de transicin adyacente a la uretra y en el lmite entre zona
de transicin y zona perifrica. Ecogrficamente se ven como zonas
hiperecoicas que dejan sombra acstica posterior. La aparicin de estas
calcificaciones solamente se traduce en un proceso degenerativo en el
interior del acino prosttico, no siendo especficos de ninguna patologa que afecte a la prstata (respuestas 1 y 4 falsas).
Las litiasis prostticas pueden ponerse de manifiesto tanto en la exploracin fsica (tacto rectal), aunque no debe obviarse nunca la posibilidad
de induracin secundaria a cncer prosttico, como en las pruebas de
imagen (Rx simple de abdomen o Ecografa transrectal). No obstante,
cursan de manera asintomtica, por lo que no son subsidiarias de tratamiento (respuestas 3 y 5 falsas). Recordemos que la litotricia extracorprea
por ondas de choque (LEOC) solamente es aplicable a aquellas litiasis
ubicadas en el trayecto ureteral, pelvis renal y cavidades caliciales.
Como primera medida a tomar siempre ante un paciente prosttico es la realizacin de un tacto rectal que nos informar sobre el

CTO Medicina C/ Nez de Balboa, 115 28006 MADRID (Espaa) Tfno.: (91) 782 43 32 / Fax: (91) 782 43 27
E-mail: secretaria@ctomedicina.com; iberocto@ctomedicina.com WEB: www.ctomedicina.com; www.iberocto.com

Comentarios TEST

Seguimiento a distancia

UROLOGA

Preparacin Examen de Seleccin 05/06 1 Vuelta


volumen y la consistencia de la prstata (respuesta 2 correcta).
Pregunta 41.- R: 2
La evaluacin inicial de un paciente prosttico incluye como 1
prueba a realizar un tacto rectal. Los posibles resultados son:
Prstata de consistencia fibroelstica: tpica de un varn joven. Desde el punto de vista mnemotcnico sera similar a la consistencia de
la eminencia tenar. El tamao sera el de una castaa, 30 cc.
Prstata adenomatosa: caracterstica de un varn adulto o anciano. Es tpica de HPB y se traduce en un crecimiento benigno de la
prstata. Mnemotcnicamente tendra una consistencia equivalente al cartlago nasal. El tamao se estratifica del I-IV, segn se trate
de una prstata pequea o grande respectivamente.
Prstata ptrea: tpica del cncer prosttico. Mnemotcnicamente
sera aquella con una consistencia similar al pulpejo del dedo
meique, si lo apretsemos fuertemente.

Comentarios TEST

La segunda prueba a realizar ante la consulta de un paciente prosttico es la determinacin de un PSA. Aunque sabemos que el PSA no es
una prueba necesaria para el diagnstico de HPB, s es una prueba
imprescindible para el diagnstico de cncer prosttico. No hay que
olvidar que el cncer prosttico puede ocasionar sntomas obstructivos
superponibles a la HPB. Es por ello que se hace obligatorio en este caso,
y puesto que el paciente presenta un prostatismo moderado, la peticin
de un PSA, para descartar la posibilidad de cncer prosttico.
Puesto que el paciente presenta clnica (prostatismo moderado), se
debe iniciar tratamiento farmacolgico junto a la determinacin del
mencionado PSA (respuestas 1 y 3 falsas, por ser incompletas, respuesta 2 verdadera).
La evaluacin inicial de un paciente con prostatismo no incluye la
realizacin de sigmoidoscopia (respuesta 4 falsa).
En el supuesto de que este paciente tuviese alguna indicacin quirrgica, primero habra que determinar cul es su patologa (HPB o
cncer de prstata) para despus poder determinar el tipo de ciruga
del que sera beneficiario. Adenomectoma o Reseccin Transuretral
Prosttica como tratamiento quirrgico de la HPB o Prostatectoma
radical como tratamiento quirrgico del cncer de prstata. A falta de
un PSA, no sabemos si el tratamiento es quirrgico, puesto que no
existe un diagnstico (respuesta 5 falsa).
Pregunta 42.- R: 4
El PSA est producido por clulas epiteliales de la prstata. Se
considera una proteasa encargada de la licuefaccin del semen. Hoy
da, es la prueba ms sensible para el diagnstico de cncer prosttico
por encima de la fosfatasa cida prosttica, que se eleva en casos de
cncer de prstata avanzados. Su rango normal es de 04 ng/ml. Sin
embargo, la especificidad para la deteccin de cncer prosttico deja
mucho que desear por varios motivos:
1. Produccin en otras localizaciones (mama, glndulas de la uretra
masculina...).
2. Situaciones que aumentan transitoriamente los niveles de PSA:
- Eyaculacin - Aumento.
- Masaje prosttico, ciclismo - Aumento.
- Cistoscopia, Ecografa Transrectal, Biopsia - Aumento.
- Retencin Aguda de Orina - Aumento.
- Infeccin prosttica - Aumento.
- Reposo - Disminucin.
- Inhibidores de la 5-alfa-reductasa - Disminucin.
- Tacto rectal - Alteracin no significativa.
Si el paciente de nuestra pregunta presenta una prostatitis aguda, su
PSA de 25,6 ng/ml se encuentra claramente artefactado, por lo que no es
valorable en ese momento. Recordemos que nosotros necesitamos determinar el PSA para descartar la posibilidad de cncer de prstata. Lo
aconsejable sera iniciar tratamiento antibitico a fin de solventar el cuadro infeccioso (quinolonas 4 semanas); una vez resuelto ste, determinaramos el PSA; por supuesto continuara con tratamiento alfa bloqueante,
a fin de aliviarle su prostatismo de base (respuesta 4 correcta).
Respuesta 1 incorrecta, por ser incompleta.
Respuestas 2 y 5 falsas, pues precisa hacer tratamiento antibitico
de su prostatitis aguda y precisa una determinacin objetiva del PSA a
M exico A rgentina
C hile U ruguay

Seguimiento a distancia

fin de valorar la posibilidad de ecografa transrectal (ETR) y toma de


biopsias.
La indicacin de biopsia prosttica viene determinada por la aparicin de un tacto rectal sospechoso de cncer de prstata, un PSA > 4
ng/ml o la visualizacin de un ndulo sospechoso al realizar una ETR.
En tanto en cuanto no poseemos un PSA valorable y el tacto rectal no es
sospechoso de CP, no est indicado realizar biopsia (respuesta 3 falsa).
Pregunta 43.- R: 5
Una vez finalizado el tratamiento, el PSA obtenido puede ser considerado como ausente de artefacto y, por lo tanto, valorable. Son
indicacin de biopsia prosttica, a fin de diagnosticar un cncer prosttico, la aparicin de:
Tacto rectal sospechoso (ptreo, indurado, nodulaciones, bordes
irregulares...).
PSA > 4 (si bien por encima de 10 ng/ml de PSA es una clara indicacin de biopsia, hay autores que en el rango de PSA entre 4-10
condicionan la realizacin de biopsia a la determinacin de otras
variables como PSA ratio, densidad de PSA, velocidad de PSA...).
Ndulo sospechoso al realizar ETR (generalmente hipoecoicos y
en zona perifrica).
Estas indicaciones absolutas de biopsias prostticas no implican
que se diagnostiquen el 100% de los CP, puesto que existen cnceres
de prstata con tacto rectal normal y PSA inferior a 4 ng/ml que pueden pasar desapercibidos al no ser subsidiarios de biopsias. Sin embargo, sera inadmisible que, a fin de que no se nos escaparan CP, se
biopsiase sistemticamente a toda la poblacin masculina por encima de 50 aos.
Con todo ello, llegamos a la conclusin de que nuestro paciente es
subsidiario de realizacin de ETR + toma de biopsias (PSA > 4 ng/ml)
(respuesta 5 verdadera -respuesta 4 falsa, pues no se realizan biopsias). El
25% de las retenciones agudas de orina (RAO) son ocasionadas por CP.
La existencia de RAO es indicacin quirrgica absoluta en el tratamiento de la HPB, pero an no hemos descartado la existencia de
posible CP (respuestas 1 y 2 falsas).
Lo primero es descartar en este paciente CP, y luego, caso de realizar
algn tratamiento, sera siempre quirrgico (respuesta 3 falsa).
Al hablar de planimetra se refiere a la obtencin del volumen
prosttico mediante la realizacin de ETR. sta es necesaria previa al
planteamiento quirrgico en la prstata.
Pregunta 44.- R: 4
Las biopsias han sido informadas como sin evidencia de malignidad, con lo cual descartamos razonablemente que nuestro paciente
tenga un CP. Asumimos que la retencin aguda es secundaria a HPB
que no ha respondido bien al tratamiento con alfabloqueantes y que
ha sufrido un proceso de descompensacin.
Sabiendo que el sujeto tiene indicacin absoluta de ciruga de la
HPB (respuestas 1, 2 y 5 falsas), tendremos que saber qu volumen
tiene la prstata, y en funcin de ello aplicaremos (ver comentario de
la pregunta siguiente):
Prstatas con un volumen <60 cc: reseccin transuretral prosttica (tratamiento endoscpico).
Prstatas con un volumen 60-80 cc: depende de las preferencias
del cirujano.
Prstatas con un volumen >80 cc: adenomectoma (Ciruga abierta).
Como el volumen de la prstata lo sabemos, puesto que se le hizo
previamente una ETR con toma de biopsias y son 85cc, el tratamiento
adecuado es la adenomectoma (respuesta 4 cierta y 3 falsa).

Pregunta 45.- R: 5
Son indicaciones de ciruga en el tratamiento de la HPB:
Retencin Aguda de Orina (RAO).
Ureterohidronefrosis bilateral 2 a uropata obstructiva infravesical.
Hematurias de repeticin de origen prosttico (Ojo: descartar tumor urotelial).
Infecciones Urinarias de repeticin, como consecuencia de orina
retenida y crecimiento de grmenes.
Litiasis vesicales como consecuencia de orina retenida y creci-

CTO Medicina C/ Nez de Balboa, 115 28006 MADRID (Espaa) Tfno.: (91) 782 43 32 / Fax: (91) 782 43 27
E-mail: secretaria@ctomedicina.com; iberocto@ctomedicina.com WEB: www.ctomedicina.com; www.iberocto.com

UR Pg. 9

miento de grmenes desdobladores de la urea.


La aparicin de una clnica irritativa severa no determina por s
misma indicacin quirrgica, ya que estos pacientes son subsidiarios
de mejora de su clnica, al administrarles tratamiento farmacolgico
(respuesta 5 falsa).

Pregunta 45. Ciruga de la HPB.


- Hematuria de repeticin.
- R etencin aguda de orina.
- ITU de repeticin.
- Hidronefrosis.
- L it ia s is .

Pregunta 46.- R: 5
Tomando valores de PSA > de 4 ng/ml como indicativos de biopsia
prosttica, nos encontramos que:
Sensibilidad
58-92%.
Especificidad
40-84%.
VPP
33-41%.
VPN
88-98%.
Segn esto, en sujetos que tengan un PSA elevado (>4), es decir con
prueba positiva, la probabilidad de que tengan cncer prosttico al realizar las biopsias es del 33-41%. Por lo tanto, la existencia de un PSA
elevado no es diagnstico de cncer de prstata (respuesta 1 verdadera).
La probabilidad de que, siendo el sujeto enfermo (tenga CP), la
prueba sea negativa (PSA <4) es igual a la tasa de falsos negativos, que
es 1- sensibilidad. De ello se deduce que TFN =1-(0,58-0,92)X 100
= 42-8%. La probabilidad de tener CP con PSA normal es del 8-42%.
Un PSA normal no descarta CP (respuesta 5 falsa).
En el eyaculado, el PSA se encuentra en forma libre, forma activa
que libera una protena conocida como seminogelina, que deriva de
las vesculas seminales, que es la causante de la disolucin del cogulo seminal (respuesta 2 verdadera).
Se ha demostrado que un nmero de tejidos extraprostticos pueden elaborar protenas similares al PSA. Se ha encontrado un PSAlike (similar al PSA) en los lquidos que rodean a las lesiones benignas
y malignas de la mama, en pncreas y glndulas que tapizan la uretra
masculina prximos al pex prosttico (respuesta 3 verdadera).
En un estudio realizado por Oesterling y cols. en 1993 sobre un
total de 361 con CP y PSA < de 10 ng/ml, tan slo 3 casos tuvieron
estudio gammagrfico positivo, aumentando exponencialmente la
positividad de la gammagrafa a medida que lo iba haciendo el nivel
de PSA (respuesta 4 verdadera).
Pregunta 47.- R: 1
Son accesibles al tacto rectal todos los estadios salvo el T1, que por
definicin es un tumor no palpable ni visible (respuesta 1 verdadera).
El PSA es un marcador de tejido prosttico cuyos niveles suelen
estar ms elevados en el CP. No obstante tambin se eleva por otras
causas benignas, lo que ha originado un intento por aumentar su
especificidad (40-84%) en el diagnstico de CP. Por este motivo, se
han empleado otros parmetros como densidad de PSA, PSA/edad,
velocidad de PSA, ratio PSA libre/ PSA total, etc. (respuesta 2 falsa).
La fosfatasa cida prosttica es un marcador especfico, pero su
elevacin suele indicar enfermedad extraprosttica, por lo que no
resulta til en el diagnstico precoz del CP, siendo su sensibilidad
deficiente para el diagnstico de CP (respuesta 3 falsa).
La ecografa abdominal no tiene gran valor en la deteccin del CP.
Es la ecografa transrectal el mtodo ms til para determinar la estadificacin local de un CP, pudiendo ofrecer suficiente informacin sobre la afectacin de la cpsula prosttica, de vesculas seminales, cuello vesical o recto (respuesta 4 falsa).
La gammagrafa sea presenta mayor sensibilidad que la radiologa
convencional para el diagnstico de metstasis seas del CP, y debe
realizarse en todo paciente al que se le sospeche tenga metstasis seas
(PSA > 20 ng/ml, fosfatasa cida prosttica elevada, dolor seo...). Sin
Pg. 10 UR

M exico A rgentina
C hile U ruguay

UROLOGA

Preparacin Examen de Seleccin 05/06 1 Vuelta

embargo, presenta una especificidad inferior a la radiologa convencional, puesto que lesiones del tipo de fracturas seas pueden producir
captacin del radioistopo, sin obligatoriamente tener que ser una
metstasis sea. La aparicin de una lesin osteocondensante (osteoblstica) en una radiografa simple, en el contexto de un paciente al que
se le sospecha un CP (tacto rectal sospechoso, PSA elevado...), es bastante especfica de metstasis sea por CP (respuesta 5 falsa).
Pregunta 48.- R: 2
La ecografa transrectal (ETR) es el mtodo ms til para determinar
la estadificacin local de un CP, pudiendo ofrecer suficiente informacin sobre la afectacin de la cpsula prosttica, de vesculas seminales, cuello vesical o recto. Aunque no existe un patrn ecogrfico
caracterstico, es de aceptacin generalizada la hipoecogenicidad de
la mayora de los cnceres de prstata. Es de vital importancia el
conocimiento de la estadificacin local del cncer prosttico, pues va
a determinar las posibilidades teraputicas que se le ofrecern al paciente y esta informacin es ofrecida gracias a la realizacin de la ETR.
La ecografa transrectal ofrece adems la posibilidad de poder dirigir
las biopsias hacia las zonas sospechosas, aportando as una constatacin histolgica evidente de la presencia de CP.
Pregunta 49.- R: 2
La indicacin de biopsia prosttica viene determinada por la aparicin de:
Un tacto rectal sospechoso de cncer de prstata (prstata indurada, bordes irregulares, ndulos palpables...).
Un PSA > 4 ng/ml.
La visualizacin de un ndulo sospechoso al realizar una ETR por
cualquier motivo.
En el paciente de esta pregunta, el hecho de tener un PSA de 3,5
ng/ml no asienta la indicacin de realizar ETR con toma de biopsias,
pero s la existencia de un tacto rectal claramente sospechoso (respuesta 2 correcta, resto falsas).
Pregunta 50.- R: 5
El empleo de la clasificacin TNM en el diagnstico del CP ha
desplazado el uso de otros sistemas de estadificacin empleados en el
CP (Clasificacin de Withmore-Jewett). Clasificacin TNM:
T1: tumor no evidente clnicamente, no palpable ni visible mediante tcnica de imagen.
- T1a: tumor detectado como hallazgo fortuito tras la realizacin
de una ciruga de HPB, con una extensin inferior al 5% del
material resecado.
- T1b: tumor detectado como hallazgo fortuito tras la realizacin
de una ciruga de HPB, con una extensin superior al 5% del
material resecado.
- T1c: tumor identificado tras la realizacin de una biopsia prosttica indicada a partir de un PSA elevado.
T2: tumor palpable o visible mediante ETR, pero confinado a la
glndula prosttica (no sobrepasa la cpsula prosttica).
- T2a: tumor que afecta a un lbulo.
- T2b: tumor que afecta a los dos lbulos.
T3: tumor palpable o visible mediante ETR, pero que se extiende
ms all de la cpsula prosttica (no organoconfinado).
- T3a: extensin extracapsular.
- T3b: extensin extracapsular con afectacin de vesculas seminales.
T4: tumor fijo que invade estructuras adyacentes (cuello vesical,
recto...).
N0: no presencia de afectacin de ganglios regionales.
N1: afectacin de ganglios regionales.
M0: no existencia de metstasis a distancia.
M1: existencia de metstasis a distancia.

CTO Medicina C/ Nez de Balboa, 115 28006 MADRID (Espaa) Tfno.: (91) 782 43 32 / Fax: (91) 782 43 27
E-mail: secretaria@ctomedicina.com; iberocto@ctomedicina.com WEB: www.ctomedicina.com; www.iberocto.com

Comentarios TEST

Seguimiento a distancia

UROLOGA

Preparacin Examen de Seleccin 05/06 1 Vuelta


El sistema Gleason valora desde el punto de vista arquitectnico la
diferenciacin glandular y el patrn de crecimiento glandular en el
estroma prosttico. Este sistema de clasificacin establece una escala
de puntuacin de 1 a 5 para valorar la diferenciacin glandular y el
patrn de crecimiento de los dos tipos histolgicos ms presentes en el
tumor. La suma de las dos escalas nos da el valor del grado final:
Valores entre 2-4: tumor bien diferenciado.
Valores entre 57: tumor moderadamente diferenciado.
Valores entre 8-10: tumor indiferenciado.
Atendiendo a la estadificacin del cncer de prstata, junto a otros
factores como edad del paciente, grado Gleason del tumor, niveles de
PSA, sintomatologa derivada de su CP; se establecen unas indicaciones estndar de tratamiento:
Estadio T1a: observacin.
Estadio T1b-T2:
- Prostatectoma radical (pacientes con una esperanza de vida >
10 aos).
- Radioterapia (pacientes con una esperanza de vida > 10 aos y
que posean un alto riesgo quirrgico o desestimen la ciruga).
Estadio T3a: radioterapia. Bloqueo hormonal (farmacolgico o
castracin quirrgica).
Estadio T3b, T4, N+, M+: bloqueo hormonal (farmacolgico o
castracin quirrgica).

Comentarios TEST

Pregunta 51.- R: 4
Las respuestas 1, 2, 3, 5 son ciertas, segn se expone en el comentario de la pregunta anterior.
La quimioterapia es un tratamiento de segunda lnea en el CP. No
supone un tratamiento estndar. Su empleo se restringe a aquellos
pacientes que desarrollan CP hormonorresistentes, lo que implica
que casi invariablemente posean a su vez metstasis a distancia o
regionales.
Por otro lado, aplicar simultneamente un bloqueo andrognico
quirrgico, como es la orquiectoma, junto a quimioterapia, solamente se sustenta en la posibilidad de que dentro del mismo tumor
existan dos tipos de poblaciones celulares, una hormonosensible y
otra hormonorresistente. Lo habitual es que, una vez confirmado el
fracaso del bloqueo andrognico, bien por ascenso paulatino de los
niveles de PSA o por evidencia clnica de progresin (aparicin de
metstasis, etc.) estando el paciente previamente bloqueado
andrognicamente, se proceda a la administracin de quimioterapia,
pero no de entrada de manera conjunta.

Pregunta 52.- R: 3
Los factores a analizar en este caso clnico son:
Edad de 68 aos con una esperanza de vida >10 aos.
Alta morbilidad (elevado riesgo quirrgico).
La presencia de un ndulo prosttico excluye que sea un estadio
T1, puesto que se palpa.
Si presenta un ndulo al tacto rectal, es indicacin de biopsia.
Conjuntamente a la realizacin de la biopsia, se hace una ecografa transrectal para dirigir las biopsias, y sta nos informa de que el
ndulo es localizado (el estadio es entonces un T2). Localizado
quiere decir que no sobrepasa la cpsula prosttica.
La biopsia da positiva para adenocarcinoma prosttico Gleason 7.

Estamos ahora en situacin de contestar la pregunta sobre qu


actitud teraputica sera la ms correcta:
EstadioT2: radioterapia (pacientes que posean contraindicaciones
quirrgicas con una esperanza de vida >10 aos). La radioterapia es
una estrategia menos agresiva que la prostatectoma radical y que
puede reportar en este tipo de pacientes con alto riesgo quirrgico y
tumores organoconfinados grandes beneficios.
Pregunta 53.- R: 3
En este paciente, si tomamos como valor nadir el de 4 ng/ml, claramente nos est indicando una mala respuesta casi segura, que se confirma con la aparicin de un PSA muy elevado, como es el de 14 ng/ml.
La existencia de metstasis en columna sacra convierte a este paM exico A rgentina
C hile U ruguay

Seguimiento a distancia

ciente con CP en un estadio M+, lo que lo hace subsidiario de tratamiento con bloqueo hormonal.
La castracin quirrgica supone el gold standard contra el cual
otros tratamientos deben ser comparados. Produce una eliminacin
de los niveles de testosterona circulante. El procedimiento quirrgico
es bien tolerado, pudindose realizar incluso con anestesia local (respuesta 3 cierta).
Pregunta 54.- R: 3
El 70-80% de los cnceres de prstata (CP) asientan sobre la zona
perifrica y solamente un 10-20% de los CP se originan en la zona de
transicin (respuesta 1 verdadera).
La gammagrafa sea se basa en la administracin de radiotrazadores. La sustancia ms empleada es el Tc99 emisor gamma puro. Constituye la tcnica ms sensible y precoz para el diagnstico de metstasis seas en el CP, con una tasa de falsos negativos del 3%, aunque
tiene el problema de la especificidad, con un 8% de falsos positivos
(respuesta 2 verdadera).
Aunque bien es cierto que la inmensa mayora de andrgenos circulantes son producidos por el testculo (clulas de Leydig bajo estmulo
de la LH) y en forma de testosterona, las zonas fascicular y reticular de la
glndula adrenal producen otros andrgenos del tipo de la
androstendiona y dehidroepiandrostendiona (bajo estmulo de la
ACTH). Los andrgenos procedentes tanto del testculo como de la
glndula adrenal son convertidos a nivel de la prstata y por accin de
la 5-alfa-reductasa en dehidrotestosterona, siendo sta el andrgeno
ms potente a nivel de la prstata. Se ha visto en ratones castrados cmo
el tamao prosttico disminua, lo que refleja que el aporte andrognico
propiciado por la glndula adrenal es insuficiente para mantener el
metabolismo prosttico normal (respuesta 3 falsa).
Se estima que entre el 90-95% de los andrgenos circulantes son
producidos por los testculos (respuesta 4 verdadera).
La castracin quirrgica supone el gold standard contra el cual
otros tratamientos deben ser comparados.
Pregunta 55.- R: 5
Las imgenes ms frecuentes son de tipo blstico, con aspecto
radiodenso sin trabeculacin sea, si bien se presentan tambin como
lesiones osteolticas, y ocurren como consecuencia de un proceso
simultneo de destruccin sea y de hiperplasia osteoblstica reactiva
(respuesta 1 verdadera).
La principal va de diseminacin y ms precoz es la linftica. Se
realiza a travs de los linfticos situados alrededor de los acinos y
conductos excretores, afectndose primariamente los ganglios obturadores seguido por los hipogstricos e ilacos. En casos muy avanzados pueden afectarse incluso ganglios por encima del diafragma (respuesta 2 correcta).
La quimioterapia no es efectiva en el CP. Se han realizado tratamientos que son mezcla de un estrgeno y una mostaza nitrogenada
(fosfato de estramustina), pero los estudios son contradictorios (respuesta 3 correcta).
En caso de compresin medular o dolor por metstasis sea, la
radioterapia paliativa (3.000 rads) puede conseguir el control local de
la enfermedad (respuesta 4 verdadera).
En pacientes no tratados previamente y con metstasis seas por
CP se puede aplicar un bloqueo andrognico, consiguiendo una
media de incremento de supervivencia de entre 23-37 meses. La supervivencia pudiera ser de 6 meses si el paciente estuviera previamente bloqueado, lo cual te indica que el tumor se est haciendo
hormonorresistente (respuesta 5 falsa).
Pregunta 56.- R: 2
En esta pregunta, se nos expone el caso de un paciente que posee
un CP con metstasis seas que producen compresin medular. En
este caso, estara contraindicada la administracin de anlogos de la
LHRH porque aumentaran inicialmente, hasta que el eje hipotlamo-hipofisario se saturase, los niveles de testosterona al aumentar los
niveles de LH, y secundariamente un estmulo sobre las clulas de
Leydig, produciendo stas testosterona y consecuentemente estimulando el crecimiento de las metstasis seas y la compresin
medular.

CTO Medicina C/ Nez de Balboa, 115 28006 MADRID (Espaa) Tfno.: (91) 782 43 32 / Fax: (91) 782 43 27
E-mail: secretaria@ctomedicina.com; iberocto@ctomedicina.com WEB: www.ctomedicina.com; www.iberocto.com

UR Pg. 11

Siempre que se empleen anlogos de la LHRH en el tratamiento


del CP se debe comenzar dando previamente antiandrgenos, a fin
de bloquear perifricamente los receptores andrognicos y evitar este
fenmeno descrito anteriormente, conocido con el nombre de flare
up (respuesta 2 verdadera).
La castracin quirrgica, administracin de antiandrgenos, el
empleo de esteroides como el dietilestilbestrol o la radioterapia, son
frenos por diferentes motivos al crecimiento de las metstasis y por
tanto se podran aplicar en este caso.
Pregunta 57.- R: 1
El carcinoma renal puede producir sndromes paraneoplsicos del
tipo de la produccin por parte del tumor de hormonas como PTH,
prostaglandinas, prolactina, renina, gonadotropinas o corticoides. El 20
% de los pacientes presentan una disfuncin heptica sin que esta implique la existencia de metstasis. Esta disfuncin heptica se conoce
con el nombre de sndrome de Stauffer (respuesta 2 verdadera).
En este tipo de pacientes se puede constatar la presencia de
anemia hasta en el 40% de los casos. Esta anemia va a ser de trastornos crnicos. En otras ocasiones lo que encontramos es una eritrocitosis 2 al efecto compresivo del tumor y a la suelta subsiguiente
de eritropoyetina al torrente sanguneo (respuesta 3 verdadera).
La trada clsica de hematuria, dolor y masa palpable slo aparece en un 10% de los casos, tratndose generalmente stos de
casos muy avanzados. La aplicacin de las pruebas de imagen en el
estudio de diversas patologas ha propiciado el aumento en el diagnstico de abundantes casos incidentales, llegando a constituir el
50% de los diagnsticos en algunas series (respuesta 4 verdadera).
Es cierto que el origen de los adenocarcinomas renales es a partir de
clulas del tbulo contorneado proximal y que el origen de los oncocitomas es a partir de clulas del aparato yuxtaglomerular. Sin embargo,
en el oncocitoma, aunque existen criterios radiolgicos para distinguirlo del adenocarcinoma, en la mayora de los casos ni stos ni la biopsia
ofrecen garantas suficientes de su benignidad, precisando una constatacin histolgica previa nefrectoma (respuesta 5 verdadera).
El adenoca. renal se origina en:

TCP

Tbulo contorneado proximal

y se ha relacionado con:

T
C
P

Tabaco
Cadmio
Plomo y poliquistosis

La extensin tumoral del cncer renal es polimorfa y puede realizarse por va directa, sangunea, linftica y excepcionalmente por va
canalicular (va urinaria). La propagacin directa por crecimiento excntrico, comprimiendo el parnquima e infiltrndole es la ms frecuente. En estados avanzados sobrepasa la cpsula renal, invadiendo
la grasa perirrenal y rganos o estructuras vecinas. La extensin va
venosa tambin es caracterstica, afectando a la vena renal ipsilateral e
incluso a la cava (respuesta 1 falsa).
Pregunta 58.- R: 4
En la actualidad se puede considerar aceptable la nefrectoma
parcial en tumores de bajo estadio (T1N0M0) y de tamao no mayor a 4 cms, incluso para pacientes con rin contralateral normal.
Existen otras indicaciones claras para realizar ciruga conservadora
como:
Enfermedad de Von Hippel-Lindau. En estos pacientes cabe esperar la aparicin de mltiples focos de cncer tanto sincrnicos
como metacrnicos. Aunque la permanencia de parnquima va a
propiciar la aparicin de nuevos tumores, se indica la tumorectoma, pues stos suelen ser de bajo grado.
Tumor renal bilateral sincrnico. Nefrectoma parcial bilateral o
bien nefrectoma radical ms nefrectoma parcial contralateral (respuesta 1 verdadera).
Pg. 12 UR

M exico A rgentina
C hile U ruguay

UROLOGA

Preparacin Examen de Seleccin 05/06 1 Vuelta

Tumor en rin nico. En paciente monorreno tanto funcional


como quirrgico.
Enfermedad parenquimatosa renal grave que pueda suponer dilisis, en caso de realizar nefrectoma.
Un 25% de los pacientes con cncer renal (CR) presentan metstasis en el momento del diagnstico, siendo las ms frecuentes y por
este orden las metstasis pulmonares, seas y hepticas. La aparicin
metacrnica de una masa renal contralateral en un paciente
nefrectomizado por CR plantea dudas de si sta se debe a una metstasis o bien se trata de un tumor renal de novo (respuesta 2 verdadera).
Enfermedad de Von Hippel-Lindau: los tumores renales con frecuencia son mltiples y bilaterales. La edad de presentacin es menor
que en la poblacin no afecta de la enfermedad (respuesta 5 verdadera).
Esclerosis Tuberosa: en el rin pueden aparecer quistes, angiomiolipomas y tumores malignos en menor frecuencia. Todos ellos
con tendencia a la multiplicidad y bilateralidad.
Se han identificado alteraciones cromosmicas que implican al
cromosoma 3 en la aparicin de tumores renales hereditarios; de
hecho, en la enfermedad de Von Hippel-Lindau existe una delecin
del cromosoma 3 (locus 3p25) (respuesta 3 verdadera).
No obstante en la mayora de los casos de tumor renal bilateral es
imposible detectar una causa gentica clara, siendo un diagnstico de
novo (respuesta 4 falsa).
Pregunta 59.- R: 5
Ver comentario de la pregunta anterior.
Pregunta 60.- R: 5
En el paciente de esta pregunta, podemos sospechar la presencia
de un tumor urotelial de vas altas (fumador y hematuria macroscpica con cogulos vermiformes), o bien un tumor de parnquima renal
(dolor en fosa lumbar que presentan el 40 % de los pacientes o
hematuria macro o microscpica, que presentan el 60% de los pacientes afectos).
El CT abdominal nos puede aportar informacin sobre la presencia de un tumor de parnquima renal, pero ante esta sospecha no es
la primera exploracin a realizar (ECO). Por otra parte, nos da menos
informacin que la UIV para dibujar la va urinaria y hacer un despistaje de tumor de va que afecte al tracto urinario superior (respuesta 1
falsa).
En este paciente con la sintomatologa descrita lo ltimo en pensar
es que se trate de un problema mdico de causa renal (glomerulopata, etc.-respuesta 2 falsa-).
La cistoscopia no valora el tracto urinario superior ni la posibilidad
de tumor de parnquima renal (respuesta 3 falsa).
Las citologas en orina orientan hacia un diagnstico de tumor de
va urinaria, pero no el de un tumor de parnquima renal (respuesta 4
falsa).
Pedir una ecografa est indicado como primera prueba a solicitar
ante la sospecha de una masa en parnquima renal. Pedir una UIV
estara indicado para el despistaje de un tumor de va que afecte al
tracto urinario superior. Y pedir un hemograma tambin est indicado, puesto que est sangrando y nos descartara anemizacin (respuesta 5 correcta).
Pregunta 61.- R: 2
La constatacin en el estudio ecogrfico de una protuberancia
slida (isoecoico con el parnquima) convierte a este quiste en
complicado. Son caractersticas de quiste complicado, la aparicin de:
Contorno irregular.
Paredes gruesas.
Calcificaciones groseras en su interior (o cualquier contenido hiper o isoecoico en la ecografa).
Realce tras la administracin de contraste (al hacer un CT).
Ante la presencia ecogrfica de un quiste con algn dato de complicacin, est indicada como primera medida la realizacin de un
CT abdominal (ver organigrama de manejo de las masas renales en

CTO Medicina C/ Nez de Balboa, 115 28006 MADRID (Espaa) Tfno.: (91) 782 43 32 / Fax: (91) 782 43 27
E-mail: secretaria@ctomedicina.com; iberocto@ctomedicina.com WEB: www.ctomedicina.com; www.iberocto.com

Comentarios TEST

Seguimiento a distancia

UROLOGA

Preparacin Examen de Seleccin 05/06 1 Vuelta


pgina siguiente).

Seguimiento a distancia

Adems, la imagen de la RMN demuestra particularmente bien la


extensin de la circulacin venosa colateral. El ultrasonido tambin
es capaz de demostrar la afectacin de la vena renal o de la cava, pero
muestra limitaciones a la hora de aportar informacin sobre la presencia de ndulos linfoides o el compromiso de otros rganos (respuesta 4 verdadera).
Pregunta 64.- R: 3
Segn la clasificacin TNM de 1997 estaramos ante un estadio
T3bNxMx o tumor que localmente se extiende macroscpicamente
en las venas renales o la cava infradiafragmtica. La vena cava es invadida entre el 4-19%, y en un 14% de stos el trombo alcanza la
aurcula derecha. En el pasado, la extensin a la vena cava se consideraba un signo ominoso. Sin embargo, desde que se comenz a aplicar
un tratamiento quirrgico agresivo, la mayora de los pacientes pueden ser curados quirrgicamente.
Si el trombo se limita a la vena renal, se procede como habitualmente en una nefrectoma radical, seccionando la vena en su porcin libre o en el ostium. Cuando protruye en el interior de la cava
se clampa la misma y se realiza cavotoma sin necesidad de controlar la vena renal contralateral. Si por el contrario el trombo es grande
y obstructivo, ser necesario la movilizacin y control completo de
la cava por arriba y por abajo con clampaje de los vasos renales
contralaterales. Si el trombo se extiende a cava supradiafragmtica,
el abordaje vendr determinado por su lmite superior. Si invade
aurcula es conveniente realizar atriotoma, lo que obliga a by-pass
cardiopulmonar. El pronstico de estos pacientes viene determinado ms por la afectacin linftica o sistmica que por la invasin
venosa vascular.

Comentarios TEST

Pregunta 61. Manejo de la masa renal descubierta accidentalmente.

Pregunta 62.- R: 3
La ecografa es la primera prueba complementaria que debe realizarse, de forma que la identificacin ecogrfica de una lesin que
cumple criterios de quiste simple (contorno liso, contenido transnico y refuerzo posterior) hace innecesarios mayores esfuerzos diagnsticos, pudiendo hacerse un seguimiento ecogrfico anual (respuesta 1
verdadera).
La deteccin mediante TAC de una masa slida o un quiste complicado en ausencia de metstasis ganglionares o sistmicas y siempre que
no se trate de un angiomiolipoma (ante la presencia de un angiomiolipoma en el CT la actitud a seguir ser el seguimiento y no actitud
quirrgica, salvo que se produzca un sangrado retroperitoneal; cuadro
conocido con el nombre de sndrome de Wnderlich) indicara, en
principio, la nefrectoma (la presencia de metstasis ganglionares o a
distancia empeora el pronstico sustancialmente. Aqu la ciruga no
tiene una intencionalidad curativa y sus indicaciones son meramente
paliativas, a fin de disminuir la mayor masa tumoral posible para posterior administracin de inmunoterapia) (respuesta 2 correcta).
Si la ecografa renal presenta unos criterios que no satisfacen las
caractersticas de quiste simple, estara indicado un CT y no una
arteriografa (respuesta 3 falsa).
En paciente monorreno tanto funcional como quirrgico, y siendo accesible quirrgicamente y de un tamao pequeo (3 cm), rene
todas las caractersticas para ser subsidiario de ciruga conservadora
(respuesta 4 verdadera).
Pregunta 63.- R: 4
Alrededor del 5% de los carcinomas renales tendrn extensin
dentro de la vena renal y pueden propagarse a travs de canales
venosos a la vena cava inferior, aurcula derecha, etc.
Generalmente no hay invasin de la pared venosa por la propagacin intraluminal del tumor. El cirujano debe ser consciente de la
verdadera extensin del tumor y particularmente conocer el lmite de
su progresin ceflica. Las imgenes de RMN son mejores que el CT y
el ultrasonido en la demostracin de la extensin venosa tumoral.
M exico A rgentina
C hile U ruguay

Pregunta 65.- R: 4
El comportamiento del carcinoma renal en cuanto a su diseminacin sistmica es caprichoso y se describen metstasis tras perodos muy prolongados (ms de 10 aos en ocasiones) libres del tumor primario. Las regresiones espontneas de estas lesiones han sido
objeto de discusin en la literatura y probablemente no afecten a
ms del 1% de los tumores metastsicos. El mecanismo de regresin
es desconocido, no se puede atribuir a la eliminacin del tumor
primario, ya que se ha descrito tambin en ausencia de tratamiento
quirrgico, y cabe buscar una explicacin inmunolgica an no
dilucidada.
En la paciente de la pregunta y habindose expuesto lo anterior, es
lgico sospechar la presencia de una metstasis tarda de un cncer
renal.
Al ser una metstasis nica, y con las reservas de todava no haber
realizado un estudio de extensin correcto ni poseer constatacin
histolgica, a fin de confirmar este dato, esta paciente sera subsidiaria de reseccin quirrgica de la metstasis nica (respuestas 2 y 3
falsas).
Nos encontramos ante un ndulo solitario que no posee caractersticas de benignidad, segn indica el enunciado (mrgenes ntidos, lesiones satlites, patrones de calcificacin que sugieran granulomas o
hamartomas). La mayora de los ndulos pulmonares solitarios corresponden a tumores pulmonares primarios, pero en este caso puede
tratarse de una metstasis tarda de un carcinoma renal. Ambas sospechas son independientes del tamao de la lesin (respuesta 5 falsa).
Lo correcto es hacer un estudio de extensin para descartar otras
metstasis o hacer evidente un tumor pulmonar primario (peticin
de un CT toracoabdominoplvico). Seguidamente y puesto que la
lesin es muy perifrica e inaccesible a la fibrobroncoscopia, realizar una PAAF, que en muchas ocasiones no es concluyente al hacer
un diagnstico citolgico pero no histolgico. Si la PAAF no es concluyente, haremos toracotoma diagnstica programando la realizacin de reseccin de metstasis nica de CR, si existe constatacin
histolgica.
Pregunta 66.- R: 4
Al realizar la ecografa se ve una imagen hiperecognica que, por
su contenido graso, sugiere como primera posibilidad que se trate de
un angiomiolipoma. Sin embargo, al ser una masa slida, requiere la
realizacin de un CT. En el CT se confirma la existencia de angiomio-

CTO Medicina C/ Nez de Balboa, 115 28006 MADRID (Espaa) Tfno.: (91) 782 43 32 / Fax: (91) 782 43 27
E-mail: secretaria@ctomedicina.com; iberocto@ctomedicina.com WEB: www.ctomedicina.com; www.iberocto.com

UR Pg. 13

lipoma (identifica las zonas de tejido adiposo que contiene, con una
baja atenuacin que oscila entre las 20 y las 100 unidades Hounsfield)
y se constata la presencia de un sangrado retroperitoneal (Sndrome
de Wnderlich).
El angiomiolipoma es un tumor benigno poco frecuente (5% de las
masas slidas renales). Est compuesto por una histologa mesodrmica (vasos sanguneos sin capa elstica o media bien definida, lo que
los hace proclives a la ruptura y sangrado, msculo liso y tejido adiposo maduro) (respuesta 1 verdadera). Cuando aparecen no asociados a
esclerosis tuberosa se suelen dar en mujeres, siendo normalmente
unilaterales y solitarios, con un tamao superior a cuando se asocian
a esclerosis tuberosa, que suelen ser mltiples y bilaterales (respuesta 4
falsa). La forma aislada incidental suele ser hoy da la forma ms frecuente de presentacin.
Las lesiones menores de 4 cm y asintomticas deben someterse a
seguimiento (respuesta 5 verdadera). Si las lesiones < de 4 cms dan sntomas, se valora la posibilidad de embolizacin selectiva para evitar sangrado, no para infartar el tumor, lo cual es tcnicamente imposible en esta
patologa (respuesta 2 verdadera). No obstante, en ocasiones no es accesible la posibilidad de embolizacin, al no poseer un equipo de radiologa vascular a nuestro alcance, optndose por la opcin quirrgica. Los
tumores asintomticos > de 4 cm deberan someterse a seguimiento ms
estrecho o plantear ciruga conservadora preventiva. Esta ltima postura
es obligada en tumores > de 4 cms y sintomticos. Si gran parte de la
unidad renal est afecta, la nefrectoma ser la nica solucin.
En las situaciones de urgencia con hemorragia masiva se abre un
amplio abanico de posibilidades (embolizacin, nefrectoma parcial
o nefrectoma total ms frecuentemente).
Pregunta 67.- R: 1
En el caso clnico de la pregunta se nos describe un sndrome de
Wnderlich con importante anemizacin de la paciente. Al realizar el
CT nos habla de zonas hipodensas, lo que pudiera estar en relacin
con la presencia de grasa procedente de un angiomiolipoma. En este
caso, que requiere cierta celeridad en la aplicacin de un tratamiento, la mayora de las veces la opcin quirrgica suele ser la alternativa
teraputica ms adoptada. La radioterapia, la quimioterapia, y por
supuesto la observacin, no son el tratamiento de eleccin en esta
situacin (respuestas 3, 4 y 5 falsas).
La linfadenectoma no est indicada en el tratamiento de las masas
renales, ni incluso en el caso de que se tratara de un cncer renal,
puesto que tiene un valor pronstico infausto, caso de ser positivas y
slamente aporta morbilidad caso de ser negativas (respuesta 2 falsa).
Al tratarse de un tumor < de 4 cm y puesto que es accesible
quirrgicamente, se intentar la nefrectoma parcial como primera
opcin (respuesta 1 verdadera).

Pregunta 67. Sndrome de Wunderlich.

Pg. 14 UR

M exico A rgentina
C hile U ruguay

UROLOGA

Preparacin Examen de Seleccin 05/06 1 Vuelta

Pregunta 68.- R: 1
Los tumores testiculares representan las neoplasias slidas ms frecuentes en los varones entre 20 y 35 aos. Histolgicamente se pueden
dividir en tumores de clulas germinales y no germinales. El primer
grupo supone el 95% del total, siendo el tumor testicular ms frecuente
el seminoma (respuesta 1 incorrecta). Tambin forman parte del grupo
de neoplasias germinales los denominados tumores no seminomatosos,
entre los que se incluyen: el coriocarcinoma, el tumor del seno endodrmico o saco vitelino (respuesta 3), el carcinoma embrionario, el
teratoma. Los tumores de clulas no germinales son muy poco frecuentes (5% del total), constituyendo un grupo heterogneo de neoplasias,
como el tumor de clulas de Leydig (que produce sintomatologa endocrina, virilizacin en el nio y signos de feminizacin en el adulto), el
de clulas de Sertoli y el gonadoblastoma, que es tpico en pacientes
con disgenesias gonadales (respuesta 2).
Los dos marcadores de tumor testicular ms importantes son la
beta-HCG (gonadotropina corinica humana, subunidad beta) y la
AFP (alfafetoprotena). La beta-HCG es producida por clulas del sincitiotrofoblasto y la AFP tiene su origen en clulas del saco vitelino
(respuestas 4 y 5).
Pregunta 69.- R: 3
Una caracterstica importante del seminoma es su radiosensibilidad,
ello va a determinar que el tratamiento de las neoplasias testiculares sea
distinto para los tumores seminomatosos que para los no seminomatosos (respuesta 1).
Debido al drenaje linftico del testculo, la primera estacin de afectacin ganglionar la constituyen los ganglios linfticos situados en el
retroperitoneo (ganglios paraarticos, interaorto-cavos, precavos, prearticos). Cuando existen adenopatas retroperitoneales se clasifica el
tumor testicular como estadio II, siendo IIA si los ganglios son menores
de 2 cm, IIB entre 2 y 5 cm, y IIC si es mayor de 5 cm (respuesta 2).
El factor de riesgo ms claramente relacionado con el cncer
testicular es la criptorquidia. Alrededor del 7-10% de los tumores
testiculares se desarrollan en pacientes que tienen una historia de
criptorquidia, siendo el seminoma el tipo de neoplasia ms comn
en estos casos. El 30% de los recin nacidos pretrmino presentan
criptorquidia, siendo la incidencia en los neonatos a trmino del 3%.
En la mayora de ellos el descenso testicular se realizar de manera
espontnea entre las seis semanas y los tres meses de vida. El 5-10%
de los tumores testiculares ocurren en el testculo contralateral descendido de manera normal (respuesta 3 incorrecta).
Los marcadores tumorales de cncer testicular (beta-HCG, AFP y
LDH) estn elevados en aproximadamente el 70% de los casos, siendo
ms frecuentes en los tumores no seminomatosos. Nos sirven no slo
para el diagnstico, sino que tambin tienen un valor pronstico y en
el seguimiento peridico tras la orquiectoma para detectar recidivas
de la enfermedad (respuesta 4).
El gonadoblastoma comprende el 0,5% de todos los tumores testiculares, y se producen de manera exclusiva en gnadas disgenticas.
Al microscopio se observan tres tipos de clulas: de Sertoli, intersticiales y germinales (respuesta 5).
Pregunta 70.- R: 3
En los testculos criptorqudicos, el riesgo relativo de malignidad es
ms alto para el testculo intraabdominal (1 en 20) que para el testculo
inguinal (1 en 80). El colocar al testculo criptorqudico en el escroto
(orquidopexia) no altera el potencial de malignizacin; sin embargo,
facilita la exploracin y la deteccin del tumor. La edad ideal para la
orquidopexia es entre el ao y los dos aos de vida (respuesta 1). Una
vez alcanzada la pubertad, los testes criptorqudicos no descendidos
probablemente deban extirparse, ya que pierden la capacidad de
espermatognesis y conservan su capacidad malignizante (respuesta 5).
Aunque entre el 8 y 25% de los pacientes tienen el antecedente de
un traumatismo testicular, ste no es un factor etiolgico claro, sino
ms bien el motivo de consulta a raz del cual se ha detectado la
neoplasia testicular (respuesta 2).
Otros factores que se han visto implicados en la gnesis de los
tumores testiculares son las hernias inguinales congnitas, la exposicin a altas temperaturas, la administracin de estrgenos intratero,
la ataxia-telangiectasia en los tumores del saco vitelino (respuesta 4).

CTO Medicina C/ Nez de Balboa, 115 28006 MADRID (Espaa) Tfno.: (91) 782 43 32 / Fax: (91) 782 43 27
E-mail: secretaria@ctomedicina.com; iberocto@ctomedicina.com WEB: www.ctomedicina.com; www.iberocto.com

Comentarios TEST

Seguimiento a distancia

UROLOGA

Preparacin Examen de Seleccin 05/06 1 Vuelta


Cuando la parotiditis afecta al nio mayor o al adulto joven, existe
riesgo de producir una orquitis vrica, la cual producir atrofia testicular. Dichos testes atrficos, como consecuencia de una orquitis urliana,
presentan mayor riesgo de malignizacin (respuesta 3 incorrecta).
Pregunta 71.- R: 3
El seminoma es el tumor testicular ms frecuente. Se han descrito
tres subtipos histolgicos: clsico (85% del total de seminomas, ms
frecuente en la cuarta dcada de la vida), anaplsico (5-10% del total,
sus clulas presentan mayor pleomorfismo nuclear) y el espermatoctico (que se suele diagnosticar en individuos mayores de 50 aos)
(respuesta 1).
El teratoma constituye el 5% del total de las neoplasias testiculares.
Contiene ms de una capa de clulas germinales en diferentes etapas
de maduracin y diferenciacin. El teratoma maduro puede tener
elementos que semejan estructuras derivadas del ectodermo, mesodermo y endodermo, mientras que el teratoma inmaduro consiste en
tejido primitivo indiferenciado (respuesta 2).
El tumor testicular ms frecuente en la infancia es el tumor del seno
endodrmico (respuesta 3 incorrecta), mientras que el ms frecuente
en el varn mayor de 60 aos es el linfoma. Las neoplasias testiculares,
cuando metastatizan, presentan, en ms de la mitad de los casos,
distinta histologa que en su localizacin primaria (respuesta 5). Ello
no ocurre con el coriocarcinoma puro, que es una lesin muy poco
frecuente y que se comporta de forma agresiva, caracterizado por
producir diseminacin hematgena temprana (respuesta 4).

Pregunta 71. Tumores testiculares.


Tipo histolgico

S e m in o m a

El m s frecuente. 3 tipos histolgicos:


Tpico.
Ana pl sico.
Esperm a toctico.

Ca . em briona rio

M eta sta tiza prem a tura m ente

Tum or del seno endodrm ico

El m s frecuente en nios

Corioca rcinom a

Comentarios TEST

Caractersticas

M eta sta tiza tem pra na m ente.


Ra ra vez es puro.

Tera tom a

Form a do por a l m enos 2 de la s 3 ca pa s


em briona ria s (ecto, m eso y endoderm o)

Tera toca rcinom a

Tera tom a + ca rcinom a em briona rio

Gona dobla stom a

Se a socia a gna da s disgentica s


(ej.: Sd de Swyer 46 xy)

T. de clula s de Sertoli

B e n ig n o s

T. de clula s de Leydig
L in f o m a

B e n ig n o s .
Pueden ser horm ona lm ente a ctivos.
El m s frecuente en >50 a os

Pregunta 72.- R: 2
La alfafetoprotena (AFP) es una glucoprotena con una masa
molecular de 70.000 daltons, que est presente en el suero fetal en
concentraciones altas, producida por el saco vitelino, hgado y tracto
gastrointestinal (respuesta 1). Tiene una vida media de 4-6 das (respuesta 2 incorrecta). Se encuentra elevada en los tumores del saco
vitelino y en el carcinoma embrionario, mientras que el seminoma
puro nunca produce AFP (respuesta 5). Se trata, no obstante, de un
marcador inespecfico, ya que existen otras situaciones en las que
puede estar incrementado, como patologas hepticas benignas, neoplasias pancreticas o en la ataxia-telangiectasia.
La beta-HCG es producida por clulas del sincitiotrofoblasto, y por
lo tanto est elevada en el 100% de los coriocarcinomas, en el 60% de
los carcinomas embrionarios y en el 7% de los seminomas (respuestas
3 y 4). Es una glucoprotena con una masa molecular de 38.000
daltons y una vida media de 24 horas. La elevacin persistente de
M exico A rgentina
C hile U ruguay

Seguimiento a distancia

cualquiera de estos dos marcadores tras el tratamiento supone la existencia de tumor residual.

Pregunta 72. Marcadores tumorales en el Ca. testicular.


Marcador

Tumor que lo produce

AF P

Carcinoma embrionario
T. del saco vitelino (seno endodrmico)

B eta-HCG

Carcinoma embrionario
Coriocarcinoma
Algunos seminomas

Pregunta 73.- R: 5
En este caso clnico se nos presenta a un varn joven con aumento
del tamao testicular de varios meses de evolucin. En la exploracin
fsica se evidencia la existencia de un probable hidrocele que impide
la palpacin del testculo. Aunque la causa ms frecuente de hidrocele
no sea un tumor testicular, debemos descartar siempre en un individuo joven con hidrocele la presencia de una neoplasia testicular
asociada. Adems, la forma de presentacin ms frecuente de un
tumor testicular es como un aumento indoloro del tamao del testculo. Para ello, el mtodo de eleccin es la ecografa testicular. Se trata
de una tcnica sencilla, fiable, que permite diferenciar entre masas
slidas y qusticas, determinar si la masa es en realidad intratesticular y
facilitar la exploracin testicular en presencia de hidrocele (respuesta
5 correcta). Si a pesar de la ecografa testicular el diagnstico no est
claro, est indicada la exploracin quirrgica por va inguinal, para
evitar la posibilidad terica de implantes tumorales en la piel del escroto. Si la exploracin confirma la presencia de una masa, es imperativo la realizacin de una orquiectoma.
Pregunta 74.- R: 5
Dentro de las neoplasias testiculares de clulas germinales, los tumores seminomatosos son los que tienen mejor pronstico, con ndices de supervivencia cercanos al 80% en la mayora de las series
(respuesta 3). De los tumores testiculares, slo el 1-2% son bilaterales,
producindose este hecho en un 50% de los casos en pacientes con
historia de criptorquidia (respuesta 4). El tratamiento inicial de todo
tipo de tumor testicular de clulas germinales es la orquiectoma total
por va inguinal (respuesta 5 incorrecta). Despus, dependiendo de la
anatoma patolgica, marcadores tumorales y estudio de extensin,
se decide la actitud teraputica. En el caso de los seminomas en estadio I (tumor confinado al testculo), se suele aadir radioterapia
infradiafragmtica de manera profilctica, para evitar diseminacin a
ganglios linfticos retroperitoneales. Si el seminoma presenta adenopatas en el retroperitoneo (estadio II), si stas son pequeas (menores
de 2 cm), el tratamiento de eleccin es la radioterapia (respuesta 2). Si
por el contrario son mayores de 2 cm, se tratan con quimioterapia.
Asimismo, el tratamiento de los tumores seminomatosos con afectacin de ganglios supradiafragmticos y/o metstasis viscerales (estadio
III), se realiza con quimioterapia (respuesta 1).
Pregunta 75.- R: 2
Nos encontramos ante un varn joven, diagnosticado de un tumor
testicular tipo seminoma en estadio III, debido a la existencia de un
ndulo metastsico en un pulmn (recordar que es precisamente el
pulmn el lugar ms frecuente de metstasis visceral en el caso de las
neoplasias testiculares). El tratamiento de los seminomas en estadios
avanzados se realiza mediante ciclos de quimioterapia, basados en el
esquema BEP (bleomicina, etopsido y cisplatino). Tras la quimioterapia se debe evaluar de nuevo al paciente, fundamentalmente mediante TAC toracoabdominoplvico y marcadores tumorales. El 90%
de los pacientes con seminoma en estadio III logran una respuesta
completa con la quimioterapia. Las masas residuales retroperitoneales despus de la quimioterapia con frecuencia son fibrosis (90%), a
menos que la masa exceda de 3 cm; bajo tales circunstancias, alrededor del 40% de los pacientes albergan seminoma residual. En tales
casos se justifica la escisin quirrgica de dichas masas residuales (respuesta 2 correcta).

CTO Medicina C/ Nez de Balboa, 115 28006 MADRID (Espaa) Tfno.: (91) 782 43 32 / Fax: (91) 782 43 27
E-mail: secretaria@ctomedicina.com; iberocto@ctomedicina.com WEB: www.ctomedicina.com; www.iberocto.com

UR Pg. 15

Pg. 16 UR

M exico A rgentina
C hile U ruguay

UROLOGA

Preparacin Examen de Seleccin 05/06 1 Vuelta

CTO Medicina C/ Nez de Balboa, 115 28006 MADRID (Espaa) Tfno.: (91) 782 43 32 / Fax: (91) 782 43 27
E-mail: secretaria@ctomedicina.com; iberocto@ctomedicina.com WEB: www.ctomedicina.com; www.iberocto.com

Comentarios TEST

Seguimiento a distancia

Vous aimerez peut-être aussi